Search Results

Search found 5942 results on 238 pages for 'total starnger'.

Page 63/238 | < Previous Page | 59 60 61 62 63 64 65 66 67 68 69 70  | Next Page >

  • Subsequent runs of rsync locally don't reduce data transferred

    - by sharakan
    I have an EC2 instance with data I want to sync to a mounted, but remote, volume, as a backup. rsync seems like the way to go with this, so as a test I took my test file (a Postgres pg_dump file) and used rsync -v to copy it to the mounted volume: [ec2-user work]$ rsync -v dump.sql.1 ../backup/dump.sql dump.sql.1 sent 821704315 bytes received 31 bytes 3416650.09 bytes/sec total size is 821603948 speedup is 1.00 Then, I ran it again, expecting to see minimal sent/received numbers because it would just be checksums. Instead... [ec2-user work]$ rsync -v dump.sql.1 ../backup/dump.sql dump.sql.1 sent 821704315 bytes received 31 bytes 3402502.47 bytes/sec total size is 821603948 speedup is 1.00 I'm new to rsync so perhaps I'm missing something, but isn't the idea that the source and destination files are checked for differences, and then a patch is generated and applied to the destination? Why is this not reducing the amount of data 'sent' to just the size of the checksums? Some background if it's relevant: the mounted volume is using s3fs, mounted with s3fs <bucketname> backup.

    Read the article

  • How much free memory should I have on my webserver?

    - by neanderslob
    I have a webserver that's currently hosting two Wordpress sites and some java-based collaboration software. The server has 2G of memory and is currently using about 1.8G of the available memory. Right now what's on here is pretty much a pilot project that's getting negligible traffic so I think it's pretty clear that I'll be needing more memory. I was wondering, if I was to release it, how I might anticipate my memory needs based on the traffic it gets. I've poked around on Google and what I've found has been a bit tenuous. Is there a good heuristic that one should use when calculating memory demands as a function of the base (no traffic) load on the server? For reference, the output of free -m can be seen below: total used free shared buffers cached Mem: 2048 1832 215 0 0 0 -/+ buffers/cache: 1832 215 Swap: 0 0 0 To me this looks like actual memory used and isn't an illusion due to caching or anything else. I figure the demands of my collaboration software will have to be experimentally tested so here's free -m without that software running: total used free shared buffers cached Mem: 2048 1109 938 0 0 0 -/+ buffers/cache: 1109 938 Swap: 0 0 0 My plan B to figure this out is to add a bunch of swap space to the server, give it some traffic and adjust according the the amount that swap gets used. I was just wondering if anyone had a good rule of thumb to estimate how much memory I should plan on in advance...or if what I'm thinking is nuts. Many thanks in advance (I'm really quite new to this).

    Read the article

  • Identifying Exchange 2010 regular process that is walking the mailbox database

    - by toongeneral
    I have an Exchange 2010 server running on a SAN-backed platform. The platform does block-level backups based on a snapshot/incremental basis, that only capture changed data. I was surprised to see a regular period of time where the data changes were happening at a high, sustained rate. Due to the way this system works, that can lead to 1.2TB of stored data per month. The regularity implied a scheduled task, but it is not a fixed interval. It is approximately every 26-32hrs. The disks were performing read operations of ~5MB/s and write operations of ~4.5MB/s, for a period of 3-4hrs. The total written data was ~55-60GB. Reading on TechNet, I am wondering if the following is causing this: http://blogs.technet.com/b/exchange/archive/2011/12/14/database-maintenance-in-exchange-2010.aspx#checksumming The somewhat restrictive thing is that the process only happens at most once every 24 hours. I was able to investigate while it was running, finding the following: the process is store.exe it is working on the mailbox database files while running, it is generating .log files (in the mailbox database folder) consistent with database changes the mailbox database is ~60GB in size, which fits with the total data changes on each iteration I have currently switched to a fixed maintenance window, as a test. It's not clear whether this is the cause, as the symptoms fit, but are not conclusive. Does anyone have any suggestions for additional troubleshooting?

    Read the article

  • Server high CPU load issue! ( Cpanel + CentOS 5)

    - by kenby
    Our server cpu load is high todays sometimes reaches to 560! .. We have the lastest Cpanel/whm and the kernel is update!while the load average is : Load Averages: 39.05 75.01 45.33 the apache log is: Current Time: Sunday, 30-Jan-2011 01:50:13 EST Restart Time: Saturday, 29-Jan-2011 21:51:20 EST Parent Server Generation: 2 Server uptime: 3 hours 58 minutes 53 seconds Total accesses: 149493 - Total Traffic: 2.4 GB CPU Usage: u9.17 s10.66 cu42.82 cs0 - .437% CPU load 10.4 requests/sec - 174.6 kB/second - 16.7 kB/request 121 requests currently being processed, 42 idle workers W_WWW.W_..W.W_W_WCWW..W...W.WWW.WWWW.WW.C_W_.W.WW.WC..W.WW.WW .W.W.W...WWWW...WW.CC.C.._W.WC.WW_WW._W....W.WWW.W.WWW.W..W WW.....WW.W_WWWWW..WCRW..WWCW.WWW__.WWWWCW_W._._WW_W...W...W _W..W..WW.W...._W..._WW.W.WWW.._W.WWW.WWW....WW_.C...W._ Scoreboard Key: "_" Waiting for Connection, "S" Starting up, "R" Reading Request, "W" Sending Reply, "K" Keepalive (read), "D" DNS Lookup, "C" Closing connection, "L" Logging, "G" Gracefully finishing, "I" Idle cleanup of worker, "." Open slot with no current process What cause this high cpu load while the apache cpu load is fine? the mysql process is also fine.. the cpu load is still high even if I stop mail-http-mysql services!

    Read the article

  • amazon ec2-medium apache requests per second terrible

    - by TheDayIsDone
    EDITED -- test running from localhost now to rule out network... i have a c1.medium using EBS. when i do an apache benchmark and i'm just printing a "hello" for the test from localhost - no database hits, it's very slow. i can repeat this test many times with the same results. any thoughts? thanks in advance. ab -n 1000 -c 100 http://localhost/home/test/ Benchmarking localhost (be patient) Completed 100 requests Completed 200 requests Completed 300 requests Completed 400 requests Completed 500 requests Completed 600 requests Completed 700 requests Completed 800 requests Completed 900 requests Completed 1000 requests Finished 1000 requests Server Software: Apache/2.2.23 Server Hostname: localhost Server Port: 80 Document Path: /home/test/ Document Length: 5 bytes Concurrency Level: 100 Time taken for tests: 25.300 seconds Complete requests: 1000 Failed requests: 0 Write errors: 0 Total transferred: 816000 bytes HTML transferred: 5000 bytes Requests per second: 39.53 [#/sec] (mean) Time per request: 2530.037 [ms] (mean) Time per request: 25.300 [ms] (mean, across all concurrent requests) Transfer rate: 31.50 [Kbytes/sec] received Connection Times (ms) min mean[+/-sd] median max Connect: 0 7 21.0 0 73 Processing: 81 2489 665.7 2500 4057 Waiting: 80 2443 654.0 2445 4057 Total: 85 2496 653.5 2500 4057 Percentage of the requests served within a certain time (ms) 50% 2500 66% 2651 75% 2842 80% 2932 90% 3301 95% 3506 98% 3762 99% 3838 100% 4057 (longest request)

    Read the article

  • High Linux loads on low CPU/memory usage

    - by user13323
    Hi. I have quite strange situation, where my CentOS 5.5 box loads are high, but the CPU and memory used are pretty low: top - 20:41:38 up 42 days, 6:14, 2 users, load average: 19.79, 21.25, 18.87 Tasks: 254 total, 1 running, 253 sleeping, 0 stopped, 0 zombie Cpu(s): 3.8%us, 0.3%sy, 0.1%ni, 95.0%id, 0.6%wa, 0.0%hi, 0.1%si, 0.0%st Mem: 4035284k total, 4008084k used, 27200k free, 38748k buffers Swap: 4208928k total, 242576k used, 3966352k free, 1465008k cached free -mt total used free shared buffers cached Mem: 3940 3910 29 0 37 1427 -/+ buffers/cache: 2445 1495 Swap: 4110 236 3873 Total: 8050 4147 3903 Iostat also shows good results: avg-cpu: %user %nice %system %iowait %steal %idle 3.83 0.13 0.41 0.58 0.00 95.05 Here is the ps aux output: USER PID %CPU %MEM VSZ RSS TTY STAT START TIME COMMAND root 1 0.0 0.0 10348 80 ? Ss 2010 2:11 init [3] root 2 0.0 0.0 0 0 ? S< 2010 0:00 [migration/0] root 3 0.0 0.0 0 0 ? SN 2010 0:00 [ksoftirqd/0] root 4 0.0 0.0 0 0 ? S< 2010 0:00 [watchdog/0] root 5 0.0 0.0 0 0 ? S< 2010 0:02 [migration/1] root 6 0.0 0.0 0 0 ? SN 2010 0:00 [ksoftirqd/1] root 7 0.0 0.0 0 0 ? S< 2010 0:00 [watchdog/1] root 8 0.0 0.0 0 0 ? S< 2010 0:02 [migration/2] root 9 0.0 0.0 0 0 ? SN 2010 0:00 [ksoftirqd/2] root 10 0.0 0.0 0 0 ? S< 2010 0:00 [watchdog/2] root 11 0.0 0.0 0 0 ? S< 2010 0:02 [migration/3] root 12 0.0 0.0 0 0 ? SN 2010 0:00 [ksoftirqd/3] root 13 0.0 0.0 0 0 ? S< 2010 0:00 [watchdog/3] root 14 0.0 0.0 0 0 ? S< 2010 0:03 [migration/4] root 15 0.0 0.0 0 0 ? SN 2010 0:00 [ksoftirqd/4] root 16 0.0 0.0 0 0 ? S< 2010 0:00 [watchdog/4] root 17 0.0 0.0 0 0 ? S< 2010 0:01 [migration/5] root 18 0.0 0.0 0 0 ? SN 2010 0:00 [ksoftirqd/5] root 19 0.0 0.0 0 0 ? S< 2010 0:00 [watchdog/5] root 20 0.0 0.0 0 0 ? S< 2010 0:11 [migration/6] root 21 0.0 0.0 0 0 ? SN 2010 0:00 [ksoftirqd/6] root 22 0.0 0.0 0 0 ? S< 2010 0:00 [watchdog/6] root 23 0.0 0.0 0 0 ? S< 2010 0:01 [migration/7] root 24 0.0 0.0 0 0 ? SN 2010 0:00 [ksoftirqd/7] root 25 0.0 0.0 0 0 ? S< 2010 0:00 [watchdog/7] root 26 0.0 0.0 0 0 ? S< 2010 0:00 [migration/8] root 27 0.0 0.0 0 0 ? SN 2010 0:00 [ksoftirqd/8] root 28 0.0 0.0 0 0 ? S< 2010 0:00 [watchdog/8] root 29 0.0 0.0 0 0 ? S< 2010 0:00 [migration/9] root 30 0.0 0.0 0 0 ? SN 2010 0:00 [ksoftirqd/9] root 31 0.0 0.0 0 0 ? S< 2010 0:00 [watchdog/9] root 32 0.0 0.0 0 0 ? S< 2010 0:08 [migration/10] root 33 0.0 0.0 0 0 ? SN 2010 0:00 [ksoftirqd/10] root 34 0.0 0.0 0 0 ? S< 2010 0:00 [watchdog/10] root 35 0.0 0.0 0 0 ? S< 2010 0:05 [migration/11] root 36 0.0 0.0 0 0 ? SN 2010 0:00 [ksoftirqd/11] root 37 0.0 0.0 0 0 ? S< 2010 0:00 [watchdog/11] root 38 0.0 0.0 0 0 ? S< 2010 0:02 [migration/12] root 39 0.0 0.0 0 0 ? SN 2010 0:00 [ksoftirqd/12] root 40 0.0 0.0 0 0 ? S< 2010 0:00 [watchdog/12] root 41 0.0 0.0 0 0 ? S< 2010 0:14 [migration/13] root 42 0.0 0.0 0 0 ? SN 2010 0:00 [ksoftirqd/13] root 43 0.0 0.0 0 0 ? S< 2010 0:00 [watchdog/13] root 44 0.0 0.0 0 0 ? S< 2010 0:04 [migration/14] root 45 0.0 0.0 0 0 ? SN 2010 0:00 [ksoftirqd/14] root 46 0.0 0.0 0 0 ? S< 2010 0:00 [watchdog/14] root 47 0.0 0.0 0 0 ? S< 2010 0:01 [migration/15] root 48 0.0 0.0 0 0 ? SN 2010 0:00 [ksoftirqd/15] root 49 0.0 0.0 0 0 ? S< 2010 0:00 [watchdog/15] root 50 0.0 0.0 0 0 ? S< 2010 0:00 [events/0] root 51 0.0 0.0 0 0 ? S< 2010 0:00 [events/1] root 52 0.0 0.0 0 0 ? S< 2010 0:00 [events/2] root 53 0.0 0.0 0 0 ? S< 2010 0:00 [events/3] root 54 0.0 0.0 0 0 ? S< 2010 0:00 [events/4] root 55 0.0 0.0 0 0 ? S< 2010 0:00 [events/5] root 56 0.0 0.0 0 0 ? S< 2010 0:00 [events/6] root 57 0.0 0.0 0 0 ? S< 2010 0:00 [events/7] root 58 0.0 0.0 0 0 ? S< 2010 0:00 [events/8] root 59 0.0 0.0 0 0 ? S< 2010 0:00 [events/9] root 60 0.0 0.0 0 0 ? S< 2010 0:00 [events/10] root 61 0.0 0.0 0 0 ? S< 2010 0:00 [events/11] root 62 0.0 0.0 0 0 ? S< 2010 0:00 [events/12] root 63 0.0 0.0 0 0 ? S< 2010 0:00 [events/13] root 64 0.0 0.0 0 0 ? S< 2010 0:00 [events/14] root 65 0.0 0.0 0 0 ? S< 2010 0:00 [events/15] root 66 0.0 0.0 0 0 ? S< 2010 0:00 [khelper] root 107 0.0 0.0 0 0 ? S< 2010 0:00 [kthread] root 126 0.0 0.0 0 0 ? S< 2010 0:00 [kblockd/0] root 127 0.0 0.0 0 0 ? S< 2010 0:03 [kblockd/1] root 128 0.0 0.0 0 0 ? S< 2010 0:01 [kblockd/2] root 129 0.0 0.0 0 0 ? S< 2010 0:00 [kblockd/3] root 130 0.0 0.0 0 0 ? S< 2010 0:05 [kblockd/4] root 131 0.0 0.0 0 0 ? S< 2010 0:00 [kblockd/5] root 132 0.0 0.0 0 0 ? S< 2010 0:00 [kblockd/6] root 133 0.0 0.0 0 0 ? S< 2010 0:00 [kblockd/7] root 134 0.0 0.0 0 0 ? S< 2010 0:00 [kblockd/8] root 135 0.0 0.0 0 0 ? S< 2010 0:02 [kblockd/9] root 136 0.0 0.0 0 0 ? S< 2010 0:00 [kblockd/10] root 137 0.0 0.0 0 0 ? S< 2010 0:00 [kblockd/11] root 138 0.0 0.0 0 0 ? S< 2010 0:04 [kblockd/12] root 139 0.0 0.0 0 0 ? S< 2010 0:00 [kblockd/13] root 140 0.0 0.0 0 0 ? S< 2010 0:00 [kblockd/14] root 141 0.0 0.0 0 0 ? S< 2010 0:00 [kblockd/15] root 142 0.0 0.0 0 0 ? S< 2010 0:00 [kacpid] root 281 0.0 0.0 0 0 ? S< 2010 0:00 [cqueue/0] root 282 0.0 0.0 0 0 ? S< 2010 0:00 [cqueue/1] root 283 0.0 0.0 0 0 ? S< 2010 0:00 [cqueue/2] root 284 0.0 0.0 0 0 ? S< 2010 0:00 [cqueue/3] root 285 0.0 0.0 0 0 ? S< 2010 0:00 [cqueue/4] root 286 0.0 0.0 0 0 ? S< 2010 0:00 [cqueue/5] root 287 0.0 0.0 0 0 ? S< 2010 0:00 [cqueue/6] root 288 0.0 0.0 0 0 ? S< 2010 0:00 [cqueue/7] root 289 0.0 0.0 0 0 ? S< 2010 0:00 [cqueue/8] root 290 0.0 0.0 0 0 ? S< 2010 0:00 [cqueue/9] root 291 0.0 0.0 0 0 ? S< 2010 0:00 [cqueue/10] root 292 0.0 0.0 0 0 ? S< 2010 0:00 [cqueue/11] root 293 0.0 0.0 0 0 ? S< 2010 0:00 [cqueue/12] root 294 0.0 0.0 0 0 ? S< 2010 0:00 [cqueue/13] root 295 0.0 0.0 0 0 ? S< 2010 0:00 [cqueue/14] root 296 0.0 0.0 0 0 ? S< 2010 0:00 [cqueue/15] root 299 0.0 0.0 0 0 ? S< 2010 0:00 [khubd] root 301 0.0 0.0 0 0 ? S< 2010 0:00 [kseriod] root 490 0.0 0.0 0 0 ? S 2010 0:00 [khungtaskd] root 493 0.1 0.0 0 0 ? S< 2010 94:48 [kswapd1] root 494 0.0 0.0 0 0 ? S< 2010 0:00 [aio/0] root 495 0.0 0.0 0 0 ? S< 2010 0:00 [aio/1] root 496 0.0 0.0 0 0 ? S< 2010 0:00 [aio/2] root 497 0.0 0.0 0 0 ? S< 2010 0:00 [aio/3] root 498 0.0 0.0 0 0 ? S< 2010 0:00 [aio/4] root 499 0.0 0.0 0 0 ? S< 2010 0:00 [aio/5] root 500 0.0 0.0 0 0 ? S< 2010 0:00 [aio/6] root 501 0.0 0.0 0 0 ? S< 2010 0:00 [aio/7] root 502 0.0 0.0 0 0 ? S< 2010 0:00 [aio/8] root 503 0.0 0.0 0 0 ? S< 2010 0:00 [aio/9] root 504 0.0 0.0 0 0 ? S< 2010 0:00 [aio/10] root 505 0.0 0.0 0 0 ? S< 2010 0:00 [aio/11] root 506 0.0 0.0 0 0 ? S< 2010 0:00 [aio/12] root 507 0.0 0.0 0 0 ? S< 2010 0:00 [aio/13] root 508 0.0 0.0 0 0 ? S< 2010 0:00 [aio/14] root 509 0.0 0.0 0 0 ? S< 2010 0:00 [aio/15] root 665 0.0 0.0 0 0 ? S< 2010 0:00 [kpsmoused] root 808 0.0 0.0 0 0 ? S< 2010 0:00 [ata/0] root 809 0.0 0.0 0 0 ? S< 2010 0:00 [ata/1] root 810 0.0 0.0 0 0 ? S< 2010 0:00 [ata/2] root 811 0.0 0.0 0 0 ? S< 2010 0:00 [ata/3] root 812 0.0 0.0 0 0 ? S< 2010 0:00 [ata/4] root 813 0.0 0.0 0 0 ? S< 2010 0:00 [ata/5] root 814 0.0 0.0 0 0 ? S< 2010 0:00 [ata/6] root 815 0.0 0.0 0 0 ? S< 2010 0:00 [ata/7] root 816 0.0 0.0 0 0 ? S< 2010 0:00 [ata/8] root 817 0.0 0.0 0 0 ? S< 2010 0:00 [ata/9] root 818 0.0 0.0 0 0 ? S< 2010 0:00 [ata/10] root 819 0.0 0.0 0 0 ? S< 2010 0:00 [ata/11] root 820 0.0 0.0 0 0 ? S< 2010 0:00 [ata/12] root 821 0.0 0.0 0 0 ? S< 2010 0:00 [ata/13] root 822 0.0 0.0 0 0 ? S< 2010 0:00 [ata/14] root 823 0.0 0.0 0 0 ? S< 2010 0:00 [ata/15] root 824 0.0 0.0 0 0 ? S< 2010 0:00 [ata_aux] root 842 0.0 0.0 0 0 ? S< 2010 0:00 [scsi_eh_0] root 843 0.0 0.0 0 0 ? S< 2010 0:00 [scsi_eh_1] root 844 0.0 0.0 0 0 ? S< 2010 0:00 [scsi_eh_2] root 845 0.0 0.0 0 0 ? S< 2010 0:00 [scsi_eh_3] root 846 0.0 0.0 0 0 ? S< 2010 0:00 [scsi_eh_4] root 847 0.0 0.0 0 0 ? S< 2010 0:00 [scsi_eh_5] root 882 0.0 0.0 0 0 ? S< 2010 0:00 [kstriped] root 951 0.0 0.0 0 0 ? S< 2010 4:24 [kjournald] root 976 0.0 0.0 0 0 ? S< 2010 0:00 [kauditd] postfix 990 0.0 0.0 54208 2284 ? S 21:19 0:00 pickup -l -t fifo -u root 1013 0.0 0.0 12676 8 ? S<s 2010 0:00 /sbin/udevd -d root 1326 0.0 0.0 90900 3400 ? Ss 14:53 0:00 sshd: root@notty root 1410 0.0 0.0 53972 2108 ? Ss 14:53 0:00 /usr/libexec/openssh/sftp-server root 2690 0.0 0.0 0 0 ? S< 2010 0:00 [kmpathd/0] root 2691 0.0 0.0 0 0 ? S< 2010 0:00 [kmpathd/1] root 2692 0.0 0.0 0 0 ? S< 2010 0:00 [kmpathd/2] root 2693 0.0 0.0 0 0 ? S< 2010 0:00 [kmpathd/3] root 2694 0.0 0.0 0 0 ? S< 2010 0:00 [kmpathd/4] root 2695 0.0 0.0 0 0 ? S< 2010 0:00 [kmpathd/5] root 2696 0.0 0.0 0 0 ? S< 2010 0:00 [kmpathd/6] root 2697 0.0 0.0 0 0 ? S< 2010 0:00 [kmpathd/7] root 2698 0.0 0.0 0 0 ? S< 2010 0:00 [kmpathd/8] root 2699 0.0 0.0 0 0 ? S< 2010 0:00 [kmpathd/9] root 2700 0.0 0.0 0 0 ? S< 2010 0:00 [kmpathd/10] root 2701 0.0 0.0 0 0 ? S< 2010 0:00 [kmpathd/11] root 2702 0.0 0.0 0 0 ? S< 2010 0:00 [kmpathd/12] root 2703 0.0 0.0 0 0 ? S< 2010 0:00 [kmpathd/13] root 2704 0.0 0.0 0 0 ? S< 2010 0:00 [kmpathd/14] root 2705 0.0 0.0 0 0 ? S< 2010 0:00 [kmpathd/15] root 2706 0.0 0.0 0 0 ? S< 2010 0:00 [kmpath_handlerd] root 2755 0.0 0.0 0 0 ? S< 2010 4:35 [kjournald] root 2757 0.0 0.0 0 0 ? S< 2010 3:38 [kjournald] root 2759 0.0 0.0 0 0 ? S< 2010 4:10 [kjournald] root 2761 0.0 0.0 0 0 ? S< 2010 4:26 [kjournald] root 2763 0.0 0.0 0 0 ? S< 2010 3:15 [kjournald] root 2765 0.0 0.0 0 0 ? S< 2010 3:04 [kjournald] root 2767 0.0 0.0 0 0 ? S< 2010 3:02 [kjournald] root 2769 0.0 0.0 0 0 ? S< 2010 2:58 [kjournald] root 2771 0.0 0.0 0 0 ? S< 2010 0:00 [kjournald] root 3340 0.0 0.0 5908 356 ? Ss 2010 2:48 syslogd -m 0 root 3343 0.0 0.0 3804 212 ? Ss 2010 0:03 klogd -x root 3430 0.0 0.0 0 0 ? S< 2010 0:50 [kondemand/0] root 3431 0.0 0.0 0 0 ? S< 2010 0:54 [kondemand/1] root 3432 0.0 0.0 0 0 ? S< 2010 0:00 [kondemand/2] root 3433 0.0 0.0 0 0 ? S< 2010 0:00 [kondemand/3] root 3434 0.0 0.0 0 0 ? S< 2010 0:00 [kondemand/4] root 3435 0.0 0.0 0 0 ? S< 2010 0:00 [kondemand/5] root 3436 0.0 0.0 0 0 ? S< 2010 0:00 [kondemand/6] root 3437 0.0 0.0 0 0 ? S< 2010 0:00 [kondemand/7] root 3438 0.0 0.0 0 0 ? S< 2010 0:00 [kondemand/8] root 3439 0.0 0.0 0 0 ? S< 2010 0:00 [kondemand/9] root 3440 0.0 0.0 0 0 ? S< 2010 0:00 [kondemand/10] root 3441 0.0 0.0 0 0 ? S< 2010 0:00 [kondemand/11] root 3442 0.0 0.0 0 0 ? S< 2010 0:00 [kondemand/12] root 3443 0.0 0.0 0 0 ? S< 2010 0:00 [kondemand/13] root 3444 0.0 0.0 0 0 ? S< 2010 0:00 [kondemand/14] root 3445 0.0 0.0 0 0 ? S< 2010 0:00 [kondemand/15] root 3461 0.0 0.0 10760 284 ? Ss 2010 3:44 irqbalance rpc 3481 0.0 0.0 8052 4 ? Ss 2010 0:00 portmap root 3526 0.0 0.0 0 0 ? S< 2010 0:00 [rpciod/0] root 3527 0.0 0.0 0 0 ? S< 2010 0:00 [rpciod/1] root 3528 0.0 0.0 0 0 ? S< 2010 0:00 [rpciod/2] root 3529 0.0 0.0 0 0 ? S< 2010 0:00 [rpciod/3] root 3530 0.0 0.0 0 0 ? S< 2010 0:00 [rpciod/4] root 3531 0.0 0.0 0 0 ? S< 2010 0:00 [rpciod/5] root 3532 0.0 0.0 0 0 ? S< 2010 0:00 [rpciod/6] root 3533 0.0 0.0 0 0 ? S< 2010 0:00 [rpciod/7] root 3534 0.0 0.0 0 0 ? S< 2010 0:00 [rpciod/8] root 3535 0.0 0.0 0 0 ? S< 2010 0:00 [rpciod/9] root 3536 0.0 0.0 0 0 ? S< 2010 0:00 [rpciod/10] root 3537 0.0 0.0 0 0 ? S< 2010 0:00 [rpciod/11] root 3538 0.0 0.0 0 0 ? S< 2010 0:00 [rpciod/12] root 3539 0.0 0.0 0 0 ? S< 2010 0:00 [rpciod/13] root 3540 0.0 0.0 0 0 ? S< 2010 0:00 [rpciod/14] root 3541 0.0 0.0 0 0 ? S< 2010 0:00 [rpciod/15] root 3563 0.0 0.0 10160 8 ? Ss 2010 0:00 rpc.statd root 3595 0.0 0.0 55180 4 ? Ss 2010 0:00 rpc.idmapd dbus 3618 0.0 0.0 21256 28 ? Ss 2010 0:00 dbus-daemon --system root 3649 0.2 0.4 563084 18796 ? S<sl 2010 179:03 mfsmount /mnt/mfs -o rw,mfsmaster=web1.ovs.local root 3702 0.0 0.0 3800 8 ? Ss 2010 0:00 /usr/sbin/acpid 68 3715 0.0 0.0 31312 816 ? Ss 2010 3:14 hald root 3716 0.0 0.0 21692 28 ? S 2010 0:00 hald-runner 68 3726 0.0 0.0 12324 8 ? S 2010 0:00 hald-addon-acpi: listening on acpid socket /var/run/acpid.socket 68 3730 0.0 0.0 12324 8 ? S 2010 0:00 hald-addon-keyboard: listening on /dev/input/event0 root 3773 0.0 0.0 62608 332 ? Ss 2010 0:00 /usr/sbin/sshd ganglia 3786 0.0 0.0 24704 988 ? Ss 2010 14:26 /usr/sbin/gmond root 3843 0.0 0.0 54144 300 ? Ss 2010 1:49 /usr/libexec/postfix/master postfix 3855 0.0 0.0 54860 1060 ? S 2010 0:22 qmgr -l -t fifo -u root 3877 0.0 0.0 74828 708 ? Ss 2010 1:15 crond root 3891 1.4 1.9 326960 77704 ? S<l 2010 896:59 mfschunkserver root 4122 0.0 0.0 18732 176 ? Ss 2010 0:10 /usr/sbin/atd root 4193 0.0 0.8 129180 35984 ? Ssl 2010 11:04 /usr/bin/ruby /usr/sbin/puppetd root 4223 0.0 0.0 18416 172 ? S 2010 0:10 /usr/sbin/smartd -q never root 4227 0.0 0.0 3792 8 tty1 Ss+ 2010 0:00 /sbin/mingetty tty1 root 4230 0.0 0.0 3792 8 tty2 Ss+ 2010 0:00 /sbin/mingetty tty2 root 4231 0.0 0.0 3792 8 tty3 Ss+ 2010 0:00 /sbin/mingetty tty3 root 4233 0.0 0.0 3792 8 tty4 Ss+ 2010 0:00 /sbin/mingetty tty4 root 4234 0.0 0.0 3792 8 tty5 Ss+ 2010 0:00 /sbin/mingetty tty5 root 4236 0.0 0.0 3792 8 tty6 Ss+ 2010 0:00 /sbin/mingetty tty6 root 5596 0.0 0.0 19368 20 ? Ss 2010 0:00 DarwinStreamingServer qtss 5597 0.8 0.9 166572 37408 ? Sl 2010 523:02 DarwinStreamingServer root 8714 0.0 0.0 0 0 ? S Jan31 0:33 [pdflush] root 9914 0.0 0.0 65612 968 pts/1 R+ 21:49 0:00 ps aux root 10765 0.0 0.0 76792 1080 ? Ss Jan24 0:58 SCREEN root 10766 0.0 0.0 66212 872 pts/3 Ss Jan24 0:00 /bin/bash root 11833 0.0 0.0 63852 1060 pts/3 S+ 17:17 0:00 /bin/sh ./launch.sh root 11834 437 42.9 4126884 1733348 pts/3 Sl+ 17:17 1190:50 /usr/bin/java -Xms128m -Xmx512m -XX:+UseConcMarkSweepGC -jar /JavaCore/JavaCore.jar root 13127 4.7 1.1 110564 46876 ? Ssl 17:18 12:55 /JavaCore/fetcher.bin root 19392 0.0 0.0 90108 3336 ? Rs 20:35 0:00 sshd: root@pts/1 root 19401 0.0 0.0 66216 1640 pts/1 Ss 20:35 0:00 -bash root 20567 0.0 0.0 90108 412 ? Ss Jan16 1:58 sshd: root@pts/0 root 20569 0.0 0.0 66084 912 pts/0 Ss Jan16 0:00 -bash root 21053 0.0 0.0 63856 28 ? S Jan30 0:00 /bin/sh /usr/bin/WowzaMediaServerd /usr/local/WowzaMediaServer/bin/setenv.sh /var/run/WowzaM root 21054 2.9 10.3 2252652 418468 ? Sl Jan30 314:25 java -Xmx1200M -server -Djava.net.preferIPv4Stack=true -Dcom.sun.management.jmxremote=true - root 21915 0.0 0.0 0 0 ? S Feb01 0:00 [pdflush] root 29996 0.0 0.0 76524 1004 pts/0 S+ 14:41 0:00 screen -x Any idea what could this be, or where I should look for more diagnostic information? Thanks.

    Read the article

  • Using a mounted NTFS share with nginx

    - by Hoff
    I have set up a local testing VM with Ubuntu Server 12.04 LTS and the LEMP stack. It's kind of an unconventional setup because instead of having all my PHP scripts on the local machine, I've mounted an NTFS share as the document root because I do my development on Windows. I had everything working perfectly up until this morning, now I keep getting a dreaded 'File not found.' error. I am almost certain this must be somehow permission related, because if I copy my site over to /var/www, nginx and php-fpm have no problems serving my PHP scripts. What I can't figure out is why all of a sudden (after a reboot of the server), no PHP files will be served but instead just the 'File not found.' error. Static files work fine, so I think it's PHP that is causing the headache. Both nginx and php-fpm are configured to run as the user www-data: root@ubuntu-server:~# ps aux | grep 'nginx\|php-fpm' root 1095 0.0 0.0 5816 792 ? Ss 11:11 0:00 nginx: master process /opt/nginx/sbin/nginx -c /etc/nginx/nginx.conf www-data 1096 0.0 0.1 6016 1172 ? S 11:11 0:00 nginx: worker process www-data 1098 0.0 0.1 6016 1172 ? S 11:11 0:00 nginx: worker process root 1130 0.0 0.4 175560 4212 ? Ss 11:11 0:00 php-fpm: master process (/etc/php5/php-fpm.conf) www-data 1131 0.0 0.3 175560 3216 ? S 11:11 0:00 php-fpm: pool www www-data 1132 0.0 0.3 175560 3216 ? S 11:11 0:00 php-fpm: pool www www-data 1133 0.0 0.3 175560 3216 ? S 11:11 0:00 php-fpm: pool www root 1686 0.0 0.0 4368 816 pts/1 S+ 11:11 0:00 grep --color=auto nginx\|php-fpm I have mounted the NTFS share at /mnt/webfiles by editing /etc/fstab and adding the following line: //192.168.0.199/c$/Websites/ /mnt/webfiles cifs username=Jordan,password=mypasswordhere,gid=33,uid=33 0 0 Where gid 33 is the www-data group and uid 33 is the user www-data. If I list the contents of one of my sites you can in fact see that they belong to the user www-data: root@ubuntu-server:~# ls -l /mnt/webfiles/nTv5-2.0 total 8 drwxr-xr-x 0 www-data www-data 0 Jun 6 19:12 app drwxr-xr-x 0 www-data www-data 0 Aug 22 19:00 assets -rwxr-xr-x 0 www-data www-data 1150 Jan 4 2012 favicon.ico -rwxr-xr-x 0 www-data www-data 1412 Dec 28 2011 index.php drwxr-xr-x 0 www-data www-data 0 Jun 3 16:44 lib drwxr-xr-x 0 www-data www-data 0 Jan 3 2012 plugins drwxr-xr-x 0 www-data www-data 0 Jun 3 16:45 vendors If I switch to the www-data user, I have no problem creating a new file on the share: root@ubuntu-server:~# su www-data $ > /mnt/webfiles/test.txt $ ls -l /mnt/webfiles | grep test\.txt -rwxr-xr-x 0 www-data www-data 0 Sep 8 11:19 test.txt There should be no problem reading or writing to the share with php-fpm running as the user www-data. When I examine the error log of nginx, it's filled with a bunch of lines that look like the following: 2012/09/08 11:22:36 [error] 1096#0: *1 FastCGI sent in stderr: "Primary script unknown" while reading response header from upstream, client: 192.168.0.199, server: , request: "GET / HTTP/1.1", upstream: "fastcgi://unix:/var/run/php5-fpm.sock:", host: "192.168.0.123" 2012/09/08 11:22:39 [error] 1096#0: *1 FastCGI sent in stderr: "Primary script unknown" while reading response header from upstream, client: 192.168.0.199, server: , request: "GET /apc.php HTTP/1.1", upstream: "fastcgi://unix:/var/run/php5-fpm.sock:", host: "192.168.0.123" It's bizarre that this was working previously and now all of sudden PHP is complaining that it can't "find" the scripts on the share. Does anybody know why this is happening? EDIT I tried editing php-fpm.conf and changing chdir to the following: chdir = /mnt/webfiles When I try and restart the php-fpm service, I get the error: Starting php-fpm [08-Sep-2012 14:20:55] ERROR: [pool www] the chdir path '/mnt/webfiles' does not exist or is not a directory This is a total load of bullshit because this directory DOES exist and is mounted! Any ls commands to list that directory work perfectly. Why the hell can't PHP-FPM see this directory?! Here are my configuration files for reference: nginx.conf user www-data; worker_processes 2; error_log /var/log/nginx/nginx.log info; pid /var/run/nginx.pid; events { worker_connections 1024; multi_accept on; } http { include fastcgi.conf; include mime.types; default_type application/octet-stream; set_real_ip_from 127.0.0.1; real_ip_header X-Forwarded-For; ## Proxy proxy_redirect off; proxy_set_header Host $host; proxy_set_header X-Real-IP $remote_addr; proxy_set_header X-Forwarded-For $proxy_add_x_forwarded_for; client_max_body_size 32m; client_body_buffer_size 128k; proxy_connect_timeout 90; proxy_send_timeout 90; proxy_read_timeout 90; proxy_buffers 32 4k; ## Compression gzip on; gzip_types text/plain text/css application/x-javascript text/xml application/xml application/xml+rss text/javascript; gzip_disable "MSIE [1-6]\.(?!.*SV1)"; ### TCP options tcp_nodelay on; tcp_nopush on; keepalive_timeout 65; sendfile on; include /etc/nginx/sites-enabled/*; } my site config server { listen 80; access_log /var/log/nginx/$host.access.log; error_log /var/log/nginx/error.log; root /mnt/webfiles/nTv5-2.0/app/webroot; index index.php; ## Block bad bots if ($http_user_agent ~* (HTTrack|HTMLParser|libcurl|discobot|Exabot|Casper|kmccrew|plaNETWORK|RPT-HTTPClient)) { return 444; } ## Block certain Referers (case insensitive) if ($http_referer ~* (sex|vigra|viagra) ) { return 444; } ## Deny dot files: location ~ /\. { deny all; } ## Favicon Not Found location = /favicon.ico { access_log off; log_not_found off; } ## Robots.txt Not Found location = /robots.txt { access_log off; log_not_found off; } if (-f $document_root/maintenance.html) { rewrite ^(.*)$ /maintenance.html last; } location ~* \.(?:ico|css|js|gif|jpe?g|png)$ { # Some basic cache-control for static files to be sent to the browser expires max; add_header Pragma public; add_header Cache-Control "max-age=2678400, public, must-revalidate"; } location / { try_files $uri $uri/ index.php; if (-f $request_filename) { break; } rewrite ^(.+)$ /index.php?url=$1 last; } location ~ \.php$ { include /etc/nginx/fastcgi.conf; fastcgi_pass unix:/var/run/php5-fpm.sock; } } php-fpm.conf ;;;;;;;;;;;;;;;;;;;;; ; FPM Configuration ; ;;;;;;;;;;;;;;;;;;;;; ; All relative paths in this configuration file are relative to PHP's install ; prefix (/opt/php5). This prefix can be dynamicaly changed by using the ; '-p' argument from the command line. ; Include one or more files. If glob(3) exists, it is used to include a bunch of ; files from a glob(3) pattern. This directive can be used everywhere in the ; file. ; Relative path can also be used. They will be prefixed by: ; - the global prefix if it's been set (-p arguement) ; - /opt/php5 otherwise ;include=etc/fpm.d/*.conf ;;;;;;;;;;;;;;;;;; ; Global Options ; ;;;;;;;;;;;;;;;;;; [global] ; Pid file ; Note: the default prefix is /opt/php5/var ; Default Value: none pid = /var/run/php-fpm.pid ; Error log file ; Note: the default prefix is /opt/php5/var ; Default Value: log/php-fpm.log error_log = /var/log/php5-fpm/php-fpm.log ; Log level ; Possible Values: alert, error, warning, notice, debug ; Default Value: notice ;log_level = notice ; If this number of child processes exit with SIGSEGV or SIGBUS within the time ; interval set by emergency_restart_interval then FPM will restart. A value ; of '0' means 'Off'. ; Default Value: 0 ;emergency_restart_threshold = 0 ; Interval of time used by emergency_restart_interval to determine when ; a graceful restart will be initiated. This can be useful to work around ; accidental corruptions in an accelerator's shared memory. ; Available Units: s(econds), m(inutes), h(ours), or d(ays) ; Default Unit: seconds ; Default Value: 0 ;emergency_restart_interval = 0 ; Time limit for child processes to wait for a reaction on signals from master. ; Available units: s(econds), m(inutes), h(ours), or d(ays) ; Default Unit: seconds ; Default Value: 0 ;process_control_timeout = 0 ; Send FPM to background. Set to 'no' to keep FPM in foreground for debugging. ; Default Value: yes ;daemonize = yes ;;;;;;;;;;;;;;;;;;;; ; Pool Definitions ; ;;;;;;;;;;;;;;;;;;;; ; Multiple pools of child processes may be started with different listening ; ports and different management options. The name of the pool will be ; used in logs and stats. There is no limitation on the number of pools which ; FPM can handle. Your system will tell you anyway :) ; Start a new pool named 'www'. ; the variable $pool can we used in any directive and will be replaced by the ; pool name ('www' here) [www] ; Per pool prefix ; It only applies on the following directives: ; - 'slowlog' ; - 'listen' (unixsocket) ; - 'chroot' ; - 'chdir' ; - 'php_values' ; - 'php_admin_values' ; When not set, the global prefix (or /opt/php5) applies instead. ; Note: This directive can also be relative to the global prefix. ; Default Value: none ;prefix = /path/to/pools/$pool ; The address on which to accept FastCGI requests. ; Valid syntaxes are: ; 'ip.add.re.ss:port' - to listen on a TCP socket to a specific address on ; a specific port; ; 'port' - to listen on a TCP socket to all addresses on a ; specific port; ; '/path/to/unix/socket' - to listen on a unix socket. ; Note: This value is mandatory. ;listen = 127.0.0.1:9000 listen = /var/run/php5-fpm.sock ; Set listen(2) backlog. A value of '-1' means unlimited. ; Default Value: 128 (-1 on FreeBSD and OpenBSD) ;listen.backlog = -1 ; List of ipv4 addresses of FastCGI clients which are allowed to connect. ; Equivalent to the FCGI_WEB_SERVER_ADDRS environment variable in the original ; PHP FCGI (5.2.2+). Makes sense only with a tcp listening socket. Each address ; must be separated by a comma. If this value is left blank, connections will be ; accepted from any ip address. ; Default Value: any ;listen.allowed_clients = 127.0.0.1 ; Set permissions for unix socket, if one is used. In Linux, read/write ; permissions must be set in order to allow connections from a web server. Many ; BSD-derived systems allow connections regardless of permissions. ; Default Values: user and group are set as the running user ; mode is set to 0666 ;listen.owner = www-data ;listen.group = www-data ;listen.mode = 0666 ; Unix user/group of processes ; Note: The user is mandatory. If the group is not set, the default user's group ; will be used. user = www-data group = www-data ; Choose how the process manager will control the number of child processes. ; Possible Values: ; static - a fixed number (pm.max_children) of child processes; ; dynamic - the number of child processes are set dynamically based on the ; following directives: ; pm.max_children - the maximum number of children that can ; be alive at the same time. ; pm.start_servers - the number of children created on startup. ; pm.min_spare_servers - the minimum number of children in 'idle' ; state (waiting to process). If the number ; of 'idle' processes is less than this ; number then some children will be created. ; pm.max_spare_servers - the maximum number of children in 'idle' ; state (waiting to process). If the number ; of 'idle' processes is greater than this ; number then some children will be killed. ; Note: This value is mandatory. pm = dynamic ; The number of child processes to be created when pm is set to 'static' and the ; maximum number of child processes to be created when pm is set to 'dynamic'. ; This value sets the limit on the number of simultaneous requests that will be ; served. Equivalent to the ApacheMaxClients directive with mpm_prefork. ; Equivalent to the PHP_FCGI_CHILDREN environment variable in the original PHP ; CGI. ; Note: Used when pm is set to either 'static' or 'dynamic' ; Note: This value is mandatory. pm.max_children = 50 ; The number of child processes created on startup. ; Note: Used only when pm is set to 'dynamic' ; Default Value: min_spare_servers + (max_spare_servers - min_spare_servers) / 2 pm.start_servers = 20 ; The desired minimum number of idle server processes. ; Note: Used only when pm is set to 'dynamic' ; Note: Mandatory when pm is set to 'dynamic' pm.min_spare_servers = 5 ; The desired maximum number of idle server processes. ; Note: Used only when pm is set to 'dynamic' ; Note: Mandatory when pm is set to 'dynamic' pm.max_spare_servers = 35 ; The number of requests each child process should execute before respawning. ; This can be useful to work around memory leaks in 3rd party libraries. For ; endless request processing specify '0'. Equivalent to PHP_FCGI_MAX_REQUESTS. ; Default Value: 0 pm.max_requests = 500 ; The URI to view the FPM status page. If this value is not set, no URI will be ; recognized as a status page. By default, the status page shows the following ; information: ; accepted conn - the number of request accepted by the pool; ; pool - the name of the pool; ; process manager - static or dynamic; ; idle processes - the number of idle processes; ; active processes - the number of active processes; ; total processes - the number of idle + active processes. ; max children reached - number of times, the process limit has been reached, ; when pm tries to start more children (works only for ; pm 'dynamic') ; The values of 'idle processes', 'active processes' and 'total processes' are ; updated each second. The value of 'accepted conn' is updated in real time. ; Example output: ; accepted conn: 12073 ; pool: www ; process manager: static ; idle processes: 35 ; active processes: 65 ; total processes: 100 ; max children reached: 1 ; By default the status page output is formatted as text/plain. Passing either ; 'html' or 'json' as a query string will return the corresponding output ; syntax. Example: ; http://www.foo.bar/status ; http://www.foo.bar/status?json ; http://www.foo.bar/status?html ; Note: The value must start with a leading slash (/). The value can be ; anything, but it may not be a good idea to use the .php extension or it ; may conflict with a real PHP file. ; Default Value: not set pm.status_path = /status ; The ping URI to call the monitoring page of FPM. If this value is not set, no ; URI will be recognized as a ping page. This could be used to test from outside ; that FPM is alive and responding, or to ; - create a graph of FPM availability (rrd or such); ; - remove a server from a group if it is not responding (load balancing); ; - trigger alerts for the operating team (24/7). ; Note: The value must start with a leading slash (/). The value can be ; anything, but it may not be a good idea to use the .php extension or it ; may conflict with a real PHP file. ; Default Value: not set ping.path = /ping ; This directive may be used to customize the response of a ping request. The ; response is formatted as text/plain with a 200 response code. ; Default Value: pong ping.response = pong ; The timeout for serving a single request after which the worker process will ; be killed. This option should be used when the 'max_execution_time' ini option ; does not stop script execution for some reason. A value of '0' means 'off'. ; Available units: s(econds)(default), m(inutes), h(ours), or d(ays) ; Default Value: 0 ;request_terminate_timeout = 0 ; The timeout for serving a single request after which a PHP backtrace will be ; dumped to the 'slowlog' file. A value of '0s' means 'off'. ; Available units: s(econds)(default), m(inutes), h(ours), or d(ays) ; Default Value: 0 ;request_slowlog_timeout = 0 ; The log file for slow requests ; Default Value: not set ; Note: slowlog is mandatory if request_slowlog_timeout is set ;slowlog = log/$pool.log.slow ; Set open file descriptor rlimit. ; Default Value: system defined value ;rlimit_files = 1024 ; Set max core size rlimit. ; Possible Values: 'unlimited' or an integer greater or equal to 0 ; Default Value: system defined value ;rlimit_core = 0 ; Chroot to this directory at the start. This value must be defined as an ; absolute path. When this value is not set, chroot is not used. ; Note: you can prefix with '$prefix' to chroot to the pool prefix or one ; of its subdirectories. If the pool prefix is not set, the global prefix ; will be used instead. ; Note: chrooting is a great security feature and should be used whenever ; possible. However, all PHP paths will be relative to the chroot ; (error_log, sessions.save_path, ...). ; Default Value: not set ;chroot = ; Chdir to this directory at the start. ; Note: relative path can be used. ; Default Value: current directory or / when chroot ;chdir = /var/www ; Redirect worker stdout and stderr into main error log. If not set, stdout and ; stderr will be redirected to /dev/null according to FastCGI specs. ; Note: on highloaded environement, this can cause some delay in the page ; process time (several ms). ; Default Value: no ;catch_workers_output = yes ; Pass environment variables like LD_LIBRARY_PATH. All $VARIABLEs are taken from ; the current environment. ; Default Value: clean env ;env[HOSTNAME] = $HOSTNAME ;env[PATH] = /usr/local/bin:/usr/bin:/bin ;env[TMP] = /tmp ;env[TMPDIR] = /tmp ;env[TEMP] = /tmp ; Additional php.ini defines, specific to this pool of workers. These settings ; overwrite the values previously defined in the php.ini. The directives are the ; same as the PHP SAPI: ; php_value/php_flag - you can set classic ini defines which can ; be overwritten from PHP call 'ini_set'. ; php_admin_value/php_admin_flag - these directives won't be overwritten by ; PHP call 'ini_set' ; For php_*flag, valid values are on, off, 1, 0, true, false, yes or no. ; Defining 'extension' will load the corresponding shared extension from ; extension_dir. Defining 'disable_functions' or 'disable_classes' will not ; overwrite previously defined php.ini values, but will append the new value ; instead. ; Note: path INI options can be relative and will be expanded with the prefix ; (pool, global or /opt/php5) ; Default Value: nothing is defined by default except the values in php.ini and ; specified at startup with the -d argument ;php_admin_value[sendmail_path] = /usr/sbin/sendmail -t -i -f [email protected] ;php_flag[display_errors] = off ;php_admin_value[error_log] = /var/log/fpm-php.www.log ;php_admin_flag[log_errors] = on ;php_admin_value[memory_limit] = 32M php_admin_value[sendmail_path] = /usr/sbin/sendmail -t -i

    Read the article

  • How Should I Generate Trade Statistics For CouchDB/Rails3 Application?

    - by James
    My Problem: I am trying to developing a web application for currency traders. The application allows traders to enter or upload information about their trades and I want to calculate a wide variety of statistics based on what the user entered. Now, normally I would use a relational database for this, but I have two requirements that don't fit well with a relational database so I am attempting to use couchdb. Those two problems are: 1) Primarily, I have a companion desktop application that users will be able to work with and replicate to the site using couchdb's awesome replication feature and 2) I would like to allow users to be able to define their own custom things to track about trades and generate results based off of what they enter. The schema less nature of couch seems perfect here, but it may end up being harder than it sounds. (I already know couch requires you to define views in advance and such so I was just planning on sticking all the custom attributes in an array and then emitting the array in the view and further processing from there.) What I Am Doing: Right now I am just emitting each trade in couch keyed by each user's system and querying with the key of the system to get an array of trades per system. Simple. I am not using a reduce function currently to calculate any stats because I couldn't figure out how to get everything I need without getting a reduce overflow error. Here is an example of rows that are getting emitted from couch: {"total_rows":134,"offset":0,"rows":[ {"id":"5b1dcd47221e160d8721feee4ccc64be", "key":["80e40ba2fa43589d57ec3f1d19db41e6","2010/05/14 04:32:37 +0000"], null, "doc":{ "_id":"5b1dcd47221e160d8721feee4ccc64be", "_rev":"1-bc9fe763e2637694df47d6f5efb58e5b", "couchrest-type":"Trade", "system":"80e40ba2fa43589d57ec3f1d19db41e6", "pair":"EUR/USD", "direction":"Buy", "entry":12600, "exit":12700, "stop_loss":12500, "profit_target":12700, "status":"Closed", "slug":"101332132375", "custom_tracking": [{"name":"signal", "value":"Pin Bar"}] "updated_at":"2010/05/14 04:32:37 +0000", "created_at":"2010/05/14 04:32:37 +0000", "result":100}} ]} In my rails 3 controller I am basically just populating an array of trades such as the one above and then extracting out the relevant data into smaller arrays that I can compute my statistics on. Here is my show action for the page that I want to display the stats and all the trades: def show @trades = Trade.by_system(:startkey => [@system.id], :endkey => [@system.id, Time.now ]) @trades.each do |trade| if trade.result > 0 @winning_trades << trade.result elsif trade.result < 0 @losing_trades << trade.result else @breakeven_trades << trade.result end if trade.direction == "Buy" @long_trades << trade.result else @short_trades << trade.result end if trade["custom_tracking"] != nil @custom_tracking << {"result" => trade.result, "variables" => trade["custom_tracking"]} end end end I am omitting some other stuff that is going on, but that is the gist of what I am doing. Then I am calculating stuff in the view layer to produce some results: <% winning_long_trades = @long_trades.reject {|trade| trade <= 0 } %> <% winning_short_trades = @short_trades.reject {|trade| trade <= 0 } %> <ul> <li>Total Trades: <%= @trades.count %></li> <li>Winners: <%= @winning_trades.size %></li> <li>Biggest Winner (Pips): <%= @winning_trades.max %></li> <li>Average Win(Pips): <%= @winning_trades.sum/@winning_trades.size %></li> <li>Losers: <%= @losing_trades.size %></li> <li>Biggest Loser (Pips): <%= @losing_trades.min %></li> <li>Average Loss(Pips): <%= @losing_trades.sum/@losing_trades.size %></li> <li>Breakeven Trades: <%= @breakeven_trades.size %></li> <li>Long Trades: <%= @long_trades.size %></li> <li>Winning Long Trades: <%= winning_long_trades.size %></li> <li>Short Trades: <%= @short_trades.size %></li> <li>Winning Short Trades: <%= winning_short_trades.size %></li> <li>Total Pips: <%= @winning_trades.sum + @losing_trades.sum %></li> <li>Win Rate (%): <%= @winning_trades.size/@trades.count.to_f * 100 %></li> </ul> This produces the following results, which aside from a few things is exactly what I want: Total Trades: 134 Winners: 70 Biggest Winner (Pips): 1488 Average Win(Pips): 440 Losers: 58 Biggest Loser (Pips): -516 Average Loss(Pips): -225 Breakeven Trades: 6 Long Trades: 125 Winning Long Trades: 67 Short Trades: 9 Winning Short Trades: 3 Total Pips: 17819 Win Rate (%): 52.23880597014925 What I Am Wondering- Finally The Actual Questions: I am starting to get really skeptical of how well this method will work when a user has 5,000 trades instead of just 134 like in this example. I anticipate most users will only have somewhere under 200 per year, but some users may have a couple thousand trades per year. Probably no more than 5,000 per year. It seems to work ok now, but the page load times are already getting a tad high for my tastes. (About 800ms to generate the page according to rails logs with about a 250ms of that spent in the view layer.) I will end up caching this page I am sure, but I still need the regenerate the page each time a trade is updated and I can't afford to have this be too slow. Sooo..... Is doing something similar here possible with a straight couchdb reduce function? I am assuming handing this off to couch would possibly help with larger data sets. I couldn't figure out how, but I suppose that doesn't mean it isn't possible. If possible, any hints will be helpful. Could I use a list function if a reduce was not available due to reduce constraints? Are couchdb list functions suitable for this type of calculations? Anyone have any idea of whether or not list functions perform well? Any hints what one would look like for the type of calculations I am trying to achieve? I thought about other options such as running the calculations at the time each trade was saved or nightly if I had to and saving the results to a statistics doc that I could then query so that all the processing was done ahead of time. I would like this to be the last resort because then I can't really filter out trades by time periods dynamically like I would really like to. (I want to have a slider that a user can slide to only show trades from that time period using the startkey and endkey in couchdb if I can.) If I should continue running the calculations inside the rails app at the time of the page view, what can I do to improve my current implementation. I am new to rails, couch and programming in general. I am sure that I could be doing something better here. Do I need to create an array for each stat or is there a better way to do that. I guess I just would really like some advice on how to tackle this problem. I want to keep the page generation time minimal since I anticipate these being some of the highest trafficked pages. My gut is that I will need to offload the statistics calculation to either couch or run the stats in advance of when they are called, but I am not sure. Lastly: Like I mentioned above, one of the primary reasons for using couch is to allow users to define their own things to track per trade. Getting the data into couch is no problem, but how would I be able to take the custom_tracking array and find how many winning trades for each named tracking attribute. If anyone can give me any hints to the possibility of doing this that would be great. Thanks a bunch. Would really appreciate any help. Willing to fork out some $$$ if someone wants to take on the problem for me. (Don't know if that is allowed on stack overflow or not.)

    Read the article

  • setting up bind to work with nsupdate (SERVFAIL)

    - by funny_ha_ha
    I'm trying to update my DNS-Server dynamically using nsupdate. Prerequisite I'm using Debian 6 on my DNS-Server and Debian 4 on my client. I created a public/private key pair using: dnssec-keygen -C -a HMAC-MD5 -b 512 -n USER sub.example.com. I then edited my named.conf.local to contain my public key and the new zone i wish to update. It now looks like this (note: I also tried allow-update { any; }; without success): zone "example.com" { type master; file "/etc/bind/primary/example.com"; notify yes; allow-update { none; }; allow-query { any; }; }; zone "sub.example.com" { type master; file "/etc/bind/primary/sub.example.com"; notify yes; allow-update { key "sub.example.com."; }; allow-query { any; }; }; key sub.example.com. { algorithm HMAC-MD5; secret "xxxx xxxx"; }; Next, I copied the private key file (key.private) to another server I want to update the zone from. I also created a textfile (update) on this server which contained the update information (note: I tried toying around with this stuff too. no success): server example.com zone sub.example.com update add sub.example.com. 86400 A 10.10.10.1 show send Now I'm trying to update the zone using: nsupdate -k key.private -v update The Problem Said command gives me the following output: Outgoing update query: ;; ->>HEADER<<- opcode: UPDATE, status: NOERROR, id: 0 ;; flags: ; ZONE: 0, PREREQ: 0, UPDATE: 0, ADDITIONAL: 0 ;; ZONE SECTION: ;sub.example.com. IN SOA ;; UPDATE SECTION: sub.example.com. 86400 IN A 10.10.10.1 update failed: SERVFAIL named debug Level 3 gives me the following information when I issue the nsupdate command on the remote server (note: I obfuscated the client IP): 06-Aug-2012 14:51:33.977 client X.X.X.X#33182: new TCP connection 06-Aug-2012 14:51:33.977 client X.X.X.X#33182: replace 06-Aug-2012 14:51:33.978 clientmgr @0x2ada3c7ee760: createclients 06-Aug-2012 14:51:33.978 clientmgr @0x2ada3c7ee760: recycle 06-Aug-2012 14:51:33.978 client @0x2ada475f1120: accept 06-Aug-2012 14:51:33.978 client X.X.X.X#33182: read 06-Aug-2012 14:51:33.978 client X.X.X.X#33182: TCP request 06-Aug-2012 14:51:33.978 client X.X.X.X#33182: request has valid signature 06-Aug-2012 14:51:33.978 client X.X.X.X#33182: recursion not available 06-Aug-2012 14:51:33.978 client X.X.X.X#33182: update 06-Aug-2012 14:51:33.978 client X.X.X.X#33182: send 06-Aug-2012 14:51:33.978 client X.X.X.X#33182: sendto 06-Aug-2012 14:51:33.979 client X.X.X.X#33182: senddone 06-Aug-2012 14:51:33.979 client X.X.X.X#33182: next 06-Aug-2012 14:51:33.979 client X.X.X.X#33182: endrequest 06-Aug-2012 14:51:33.979 client X.X.X.X#33182: read 06-Aug-2012 14:51:33.986 client X.X.X.X#33182: next 06-Aug-2012 14:51:33.986 client X.X.X.X#33182: request failed: end of file 06-Aug-2012 14:51:33.986 client X.X.X.X#33182: endrequest 06-Aug-2012 14:51:33.986 client X.X.X.X#33182: closetcp But it doesn't do anything. The zone isn't updated, nor does my nsupdate change anything. I'm not sure if the file /etc/bind/primary/sub.example.com should exist prior to the first update or not. I tried it without the file, with an empty file and with a pre-configured zone file. Without success. The sparse information I found on the net pointed me towards file and folder permissions regarding the bind working directory, so I changed the permissions of both /etc/bind and /var/cache/bind (which is the home dir of my "bind" user). I'm not a 100% sure if the permissions are correct.. but it looks good to me: ls -lah /var/cache/bind/ total 224K drwxrwxr-x 2 bind bind 4.0K Aug 6 03:13 . drwxr-xr-x 12 root root 4.0K Jul 21 11:27 .. -rw-r--r-- 1 bind bind 211K Aug 6 03:21 named.run ls -lah /etc/bind/ total 72K drwxr-sr-x 3 bind bind 4.0K Aug 6 14:41 . drwxr-xr-x 87 root root 4.0K Jul 30 01:24 .. -rw------- 1 bind bind 125 Aug 6 02:54 key.public -rw------- 1 bind bind 156 Aug 6 02:54 key.private -rw-r--r-- 1 bind bind 2.5K Aug 6 03:07 bind.keys -rw-r--r-- 1 bind bind 237 Aug 6 03:07 db.0 -rw-r--r-- 1 bind bind 271 Aug 6 03:07 db.127 -rw-r--r-- 1 bind bind 237 Aug 6 03:07 db.255 -rw-r--r-- 1 bind bind 353 Aug 6 03:07 db.empty -rw-r--r-- 1 bind bind 270 Aug 6 03:07 db.local -rw-r--r-- 1 bind bind 3.0K Aug 6 03:07 db.root -rw-r--r-- 1 bind bind 493 Aug 6 03:32 named.conf -rw-r--r-- 1 bind bind 490 Aug 6 03:07 named.conf.default-zones -rw-r--r-- 1 bind bind 1.2K Aug 6 14:18 named.conf.local -rw-r--r-- 1 bind bind 666 Jul 29 22:51 named.conf.options drwxr-sr-x 2 bind bind 4.0K Aug 6 03:57 primary/ -rw-r----- 1 root bind 77 Mar 19 02:57 rndc.key -rw-r--r-- 1 bind bind 1.3K Aug 6 03:07 zones.rfc1918 ls -lah /etc/bind/primary/ total 20K drwxr-sr-x 2 bind bind 4.0K Aug 6 03:57 . drwxr-sr-x 3 bind bind 4.0K Aug 6 14:41 .. -rw-r--r-- 1 bind bind 356 Jul 30 00:45 example.com

    Read the article

  • Debugging matchit plugin in vim (under Cygwin)

    - by system PAUSE
    The "matchit" plugin for vim is supposed to allow you to use the % key to jump between matching start/end tags when editing HTML, as well as /* and */ comment delimiters when editing other kinds of code. I've followed the exact instructions in ":help matchit", but % still doesn't work for me. It seems silly to ask "Why doesn't this work?" so instead I'm asking How can I diagnose the problem? Pointers to references are welcome, but specific vim-plugin-debugging techniques are preferred. Here is the ~/.vim directory: $ ls -ltaGR ~/.vim /cygdrive/y/.vim: total 0 drwxr-xr-x 1 spause 0 Sep 17 13:20 .. drwxr-xr-x 1 spause 0 Sep 16 13:59 doc drwxr-xr-x 1 spause 0 Sep 16 13:58 . drwxr-xr-x 1 spause 0 Sep 16 13:58 plugin /cygdrive/y/.vim/doc: total 24 -rw-r--r-- 1 spause 1961 Sep 16 13:59 tags drwxr-xr-x 1 spause 0 Sep 16 13:59 . -rw-r--r-- 1 spause 19303 Sep 16 13:58 matchit.txt drwxr-xr-x 1 spause 0 Sep 16 13:58 .. /cygdrive/y/.vim/plugin: total 32 drwxr-xr-x 1 spause 0 Sep 16 13:58 .. -rw-r--r-- 1 spause 30714 Sep 16 13:58 matchit.vim drwxr-xr-x 1 spause 0 Sep 16 13:58 . I am running vim 7.2 under Cygwin (installed Fall 2008). cygcheck shows: 1829k 2008/06/12 C:\cygwin\bin\cygwin1.dll Cygwin DLL version info: DLL version: 1.5.25 DLL epoch: 19 DLL bad signal mask: 19005 DLL old termios: 5 DLL malloc env: 28 API major: 0 API minor: 156 Shared data: 4 DLL identifier: cygwin1 Mount registry: 2 Cygnus registry name: Cygnus Solutions Cygwin registry name: Cygwin Program options name: Program Options Cygwin mount registry name: mounts v2 Cygdrive flags: cygdrive flags Cygdrive prefix: cygdrive prefix Cygdrive default prefix: Build date: Thu Jun 12 19:34:46 CEST 2008 CVS tag: cr-0x5f1 Shared id: cygwin1S4 In vim, :set shows: --- Options --- autoindent fileformat=dos shiftwidth=3 background=dark filetype=html syntax=html cedit=^F scroll=24 tabstop=3 expandtab shelltemp textmode viminfo='20,<50,s10,h Notably, the syntax and filetype are both recognized as HTML. (The syntax colouring is just fine.) If additional info is needed, please comment. UPDATE: Per answer by too much php: After trying vim -V1, I had changed my .vimrc to include a line set nocp so the compatible option is not on. :let loadad_matchit loaded_matchit #1 :set runtimepath? runtimepath=~/.vim,/usr/share/vim/vimfiles,/usr/share/vim/vim72,/usr/share/vim/vimfiles/after,~/.vim/after (~ is /cygdrive/y) Per answer by michael: :scriptnames 1: /cygdrive/y/.vimrc 2: /usr/share/vim/vim72/syntax/syntax.vim 3: /usr/share/vim/vim72/syntax/synload.vim 4: /usr/share/vim/vim72/syntax/syncolor.vim 5: /usr/share/vim/vim72/filetype.vim 6: /usr/share/vim/vim72/colors/evening.vim 7: /cygdrive/y/.vim/plugin/matchit.vim 8: /cygdrive/y/.vim/plugin/python_match.vim 9: /usr/share/vim/vim72/plugin/getscriptPlugin.vim 10: /usr/share/vim/vim72/plugin/gzip.vim 11: /usr/share/vim/vim72/plugin/matchparen.vim 12: /usr/share/vim/vim72/plugin/netrwPlugin.vim 13: /usr/share/vim/vim72/plugin/rrhelper.vim 14: /usr/share/vim/vim72/plugin/spellfile.vim 15: /usr/share/vim/vim72/plugin/tarPlugin.vim 16: /usr/share/vim/vim72/plugin/tohtml.vim 17: /usr/share/vim/vim72/plugin/vimballPlugin.vim 18: /usr/share/vim/vim72/plugin/zipPlugin.vim 19: /usr/share/vim/vim72/syntax/html.vim 20: /usr/share/vim/vim72/syntax/javascript.vim 21: /usr/share/vim/vim72/syntax/vb.vim 22: /usr/share/vim/vim72/syntax/css.vim Note that matchit.vim, html.vim, tohtml.vim, css.vim, and javascript.vim are all present. :echo b:match_words E121: Undefined variable: b:match_words E15: Invalid expression: b:match_words Hm, this looks highly relevant. I'm now looking through :help matchit-debug to find out how to fix b:match_words.

    Read the article

  • using 'ar' tool in Alchemy

    - by paleozogt
    I've found that if you specify a path to Alchemy's 'ar' tool, it won't create the 'l.bc' file necessary to link the library. For example, here is the case when I don't specify a path (it works): asimmons-mac:test asimmons$ echo 'int main() { return 42; }' > testmain.cpp asimmons-mac:test asimmons$ echo 'int test1() { return -1; }' > test1.cpp asimmons-mac:test asimmons$ echo 'int test2() { return 1; }' > test2.cpp asimmons-mac:test asimmons$ g++ -c testmain.cpp asimmons-mac:test asimmons$ g++ -c test1.cpp asimmons-mac:test asimmons$ g++ -c test2.cpp asimmons-mac:test asimmons$ ar cr libtest.a test1.o test2.o asimmons-mac:test asimmons$ g++ testmain.cpp libtest.a llvm-ld, "-o=".(98237.achacks.o = "98237.achacks.exe"), -disable-opt -internalize-public-api-list=_start,malloc,free,__adddi3,__anddi3,__ashldi3,__ashrdi3,__cmpdi2,__divdi3,__fixdfdi,__fixsfdi,__fixunsdfdi,__fixunssfdi,__floatdidf,__floatdisf,__floatunsdidf,__iordi3,__lshldi3,__lshrdi3,__moddi3,__muldi3,__negdi2,__one_cmpldi2,__qdivrem,__adddi3,__anddi3,__ashldi3,__ashrdi3,__cmpdi2,__divdi3,__qdivrem,__fixdfdi,__fixsfdi,__fixunsdfdi,__fixunssfdi,__floatdidf,__floatdisf,__floatunsdidf,__iordi3,__lshldi3,__lshrdi3,__moddi3,__muldi3,__negdi2,__one_cmpldi2,__subdi3,__ucmpdi2,__udivdi3,__umoddi3,__xordi3,__subdi3,__ucmpdi2,__udivdi3,__umoddi3,__xordi3,__error /Users/asimmons/Development/alchemy-darwin-v0.5a/avm2-libc/lib/avm2-libc.l.bc /Users/asimmons/Development/alchemy-darwin-v0.5a/avm2-libc/lib/avm2-libstdc++.l.bc, test.l.bc 98237.achacks.o 98237.achacks.swf, 5593510 bytes written asimmons-mac:test asimmons$ ls -l total 10992 -rwxr-xr-x 1 asimmons staff 5593575 Apr 9 17:44 a.exe -rw------- 1 asimmons staff 1284 Apr 9 17:43 libtest.a -rw-r--r-- 1 asimmons staff 672 Apr 9 17:43 test.l.bc -rw-r--r-- 1 asimmons staff 27 Apr 9 17:43 test1.cpp -rwxr-xr-x 1 asimmons staff 536 Apr 9 17:43 test1.o -rw-r--r-- 1 asimmons staff 26 Apr 9 17:43 test2.cpp -rwxr-xr-x 1 asimmons staff 536 Apr 9 17:43 test2.o -rw-r--r-- 1 asimmons staff 26 Apr 9 17:43 testmain.cpp -rwxr-xr-x 1 asimmons staff 552 Apr 9 17:43 testmain.o asimmons-mac:test asimmons$ And here is an example where I do specify a path (it doesn't work). I try to tell 'ar' to put the library under 'lib' and then link to lib/libtest.a: asimmons-mac:test asimmons$ mkdir lib asimmons-mac:test asimmons$ echo 'int main() { return 42; }' > testmain.cpp asimmons-mac:test asimmons$ echo 'int test1() { return -1; }' > test1.cpp asimmons-mac:test asimmons$ echo 'int test2() { return 1; }' > test2.cpp asimmons-mac:test asimmons$ g++ -c testmain.cpp asimmons-mac:test asimmons$ g++ -c test1.cpp asimmons-mac:test asimmons$ g++ -c test2.cpp asimmons-mac:test asimmons$ ar cr lib/libtest.a test1.o test2.o asimmons-mac:test asimmons$ g++ testmain.cpp lib/libtest.a llvm-ld, "-o=".(98638.achacks.o = "98638.achacks.exe"), -disable-opt -internalize-public-api-list=_start,malloc,free,__adddi3,__anddi3,__ashldi3,__ashrdi3,__cmpdi2,__divdi3,__fixdfdi,__fixsfdi,__fixunsdfdi,__fixunssfdi,__floatdidf,__floatdisf,__floatunsdidf,__iordi3,__lshldi3,__lshrdi3,__moddi3,__muldi3,__negdi2,__one_cmpldi2,__qdivrem,__adddi3,__anddi3,__ashldi3,__ashrdi3,__cmpdi2,__divdi3,__qdivrem,__fixdfdi,__fixsfdi,__fixunsdfdi,__fixunssfdi,__floatdidf,__floatdisf,__floatunsdidf,__iordi3,__lshldi3,__lshrdi3,__moddi3,__muldi3,__negdi2,__one_cmpldi2,__subdi3,__ucmpdi2,__udivdi3,__umoddi3,__xordi3,__subdi3,__ucmpdi2,__udivdi3,__umoddi3,__xordi3,__error /Users/asimmons/Development/alchemy-darwin-v0.5a/avm2-libc/lib/avm2-libc.l.bc /Users/asimmons/Development/alchemy-darwin-v0.5a/avm2-libc/lib/avm2-libstdc++.l.bc, lib/test.l.bc 98638.achacks.o llvm-ld: error: Cannot find linker input 'lib/test.l.bc' asimmons-mac:test asimmons$ ls -l total 56 -rw-r--r-- 1 asimmons staff 552 Apr 9 17:46 98638.achacks.o drwxr-xr-x 3 asimmons staff 102 Apr 9 17:46 lib -rw-r--r-- 1 asimmons staff 27 Apr 9 17:45 test1.cpp -rwxr-xr-x 1 asimmons staff 536 Apr 9 17:46 test1.o -rw-r--r-- 1 asimmons staff 26 Apr 9 17:45 test2.cpp -rwxr-xr-x 1 asimmons staff 536 Apr 9 17:46 test2.o -rw-r--r-- 1 asimmons staff 26 Apr 9 17:45 testmain.cpp -rwxr-xr-x 1 asimmons staff 552 Apr 9 17:45 testmain.o asimmons-mac:test asimmons$ ls -l lib/ total 8 -rw------- 1 asimmons staff 1284 Apr 9 17:46 libtest.a asimmons-mac:test asimmons$ but the linker errors out because it can't find lib/test.l.bc. Notice how in the first example, 'test.l.bc' was generated alongside libtest.a. But in the second example test.l.bc was not generated. Where did it go? This is a contrived example, but in the project I'm trying to build with alchemy the make scripts generate libraries in full paths and then refer to them that way. It seems that alchemy's 'ar' tool is broken if you try to generate a library anywhere other than '.'. Has anyone else seen this? Is there a workaround? fyi, I've also posted this question on the Alchemy formus.

    Read the article

  • Reversing page navigation on PHP

    - by ilnur777
    Can anyone help me with reversing this PHP page navigation? Current script setting shows this format: [0] | 1 | 2 | 3 | 4 | 5 | 6 | 7 | 8 ... 14 • Forward • End But I really need it to reverse for this format: [14] | 13 | 12 | 11| 10 | 9 | 8 | 7 | 6 ... 0 • Back • Start Here is the PHP code: <? $onpage = 10; // on page function page(){ if(empty($_GET["page"])){ $page = 0; } else { if(!is_numeric($_GET["page"])) die("Bad page number!"); $page = $_GET["page"]; } return $page; } function navigation($onpage, $page){ //---------------- $countt = 150; $cnt=$countt; // total amount of entries $rpp=$onpage; // total entries per page $rad=4; // amount of links to show near current page (2 left + 2 right + current page = total 5) $links=$rad*2+1; $pages=ceil($cnt/$rpp); if ($page>0) { echo "<a href=\"?page=0\"><<< Start</a> <font color='#CCCCCC'>•</font> <a href=\"?page=".($page-1)."\">< Back</a> <font color='#CCCCCC'>•</font>"; } $start=$page-$rad; if ($start>$pages-$links) { $start=$pages-$links; } if ($start<0) { $start=0; } $end=$start+$links; if ($end>$pages) { $end=$pages; } for ($i=$start; $i<$end; $i++) { echo " "; if ($i==$page) { echo "["; } else { echo "<a href=\"?page=$i\">"; } echo $i; if ($i==$page) { echo "]"; } else { echo "</a>"; } if ($i!=($end-1)) { echo " <font color='#CCCCCC'>|</font>"; } } if ($pages>$links&&$page<($pages-$rad-1)) { echo " ... <a href=\"?page=".($pages-1)."\">".($pages-1)."</a>"; } if ($page<$pages-1) { echo " <font color='#CCCCCC'>•</font> <a href=\"?page=".($page+1)."\">Forward ></a> <font color='#CCCCCC'>•</font> <a href=\"?page=".($pages-1)."\">End >>></a>"; } } $page = page(); // detect page $navigation = navigation($onpage, $page); // detect navigation ?>

    Read the article

  • wpf window refresh works at first, then stops

    - by mcl
    I've got a routine that grabs a list of all images in a directory, then runs an MD5 digest on all of them. Since this takes a while to do, I pop up a window with a progress bar. The progress bar is updated by a lambda that I pass in to the long-running routine. The first problem was that the progress window was never updated (which is normal in WPF I guess). Since WPF lacks a Refresh() command I fixed this with a call to Dispatcher.Invoke(). Now the progress bar is updated for a while, then the window stops being updated. The long-running work does eventually finish and the windows go back to normal. I have already tried a BackgroundWorker and quickly became frustrated by a threading issue related to an event triggered by the long-running process. So if that's really the best solution and I just need to learn the paradigm better, please say so. But I'd be really much happier with the approach I've got here, except that it stops updating after a bit (for example, in a folder with 1000 files, it might update for 50-100 files, then "hang"). The UI does not need to be responsive during this activity, except to report on progress. Anyway, here's the code. First the progress window itself: public partial class ProgressWindow : Window { public ProgressWindow(string title, string supertext, string subtext) { InitializeComponent(); this.Title = title; this.SuperText.Text = supertext; this.SubText.Text = subtext; } internal void UpdateProgress(int count, int total) { this.ProgressBar.Maximum = Convert.ToDouble(total); this.ProgressBar.Value = Convert.ToDouble(count); this.SubText.Text = String.Format("{0} of {1} finished", count, total); this.Dispatcher.Invoke(DispatcherPriority.Render, EmptyDelegate); } private static Action EmptyDelegate = delegate() { }; } <Window x:Class="Pixort.ProgressWindow" xmlns="http://schemas.microsoft.com/winfx/2006/xaml/presentation" xmlns:x="http://schemas.microsoft.com/winfx/2006/xaml" Title="Pixort Progress" Height="128" Width="256" WindowStartupLocation="CenterOwner" WindowStyle="SingleBorderWindow" ResizeMode="NoResize"> <DockPanel> <TextBlock DockPanel.Dock="Top" x:Name="SuperText" TextAlignment="Left" Padding="6"></TextBlock> <TextBlock DockPanel.Dock="Bottom" x:Name="SubText" TextAlignment="Right" Padding="6"></TextBlock> <ProgressBar x:Name="ProgressBar" Height="24" Margin="6"/> </DockPanel> </Window> The long running method (in Gallery.cs): public void ImportFolder(string folderPath, Action<int, int> progressUpdate) { string[] files = this.FileIO.GetFiles(folderPath); for (int i = 0; i < files.Length; i++) { // do stuff with the file if (null != progressUpdate) { progressUpdate.Invoke(i + 1, files.Length); } } } Which is called thusly: ProgressWindow progress = new ProgressWindow("Import Folder Progress", String.Format("Importing {0}", folder), String.Empty); progress.Show(); this.Gallery.ImportFolder(folder, ((c, t) => progress.UpdateProgress(c, t))); progress.Close();

    Read the article

  • Convert NSData to primitive variable with ieee-754 or twos-complement ?

    - by William GILLARD
    Hi every one. I am new programmer in Obj-C and cocoa. Im a trying to write a framework which will be used to read a binary files (Flexible Image Transport System or FITS binary files, usually used by astronomers). The binary data, that I am interested to extract, can have various formats and I get its properties by reading the header of the FITS file. Up to now, I manage to create a class to store the content of the FITS file and to isolate the header into a NSString object and the binary data into a NSData object. I also manage to write method which allow me to extract the key values from the header that are very valuable to interpret the binary data. I am now trying to convert the NSData object into a primitive array (array of double, int, short ...). But, here, I get stuck and would appreciate any help. According to the documentation I have about the FITS file, I have 5 possibilities to interpret the binary data depending on the value of the BITPIX key: BITPIX value | Data represented 8 | Char or unsigned binary int 16 | 16-bit two's complement binary integer 32 | 32-bit two's complement binary integer 64 | 64-bit two's complement binary integer -32 | IEEE single precision floating-point -64 | IEEE double precision floating-point I already write the peace of code, shown bellow, to try to convert the NSData into a primitive array. // self reefer to my FITS class which contain a NSString object // with the content of the header and a NSData object with the binary data. -(void*) GetArray { switch (BITPIX) { case 8: return [self GetArrayOfUInt]; break; case 16: return [self GetArrayOfInt]; break; case 32: return [self GetArrayOfLongInt]; break; case 64: return [self GetArrayOfLongLong]; break; case -32: return [self GetArrayOfFloat]; break; case -64: return [self GetArrayOfDouble]; break; default: return NULL; } } // then I show you the method to convert the NSData into a primitive array. // I restrict my example to the case of 'double'. Code is similar for other methods // just change double by 'unsigned int' (BITPIX 8), 'short' (BITPIX 16) // 'int' (BITPIX 32) 'long lon' (BITPIX 64), 'float' (BITPIX -32). -(double*) GetArrayOfDouble { int Nelements=[self NPIXEL]; // Metod to extract, from the header // the number of element into the array NSLog(@"TOTAL NUMBER OF ELEMENTS [%i]\n",Nelements); //CREATE THE ARRAY double (*array)[Nelements]; // Get the total number of bits in the binary data int Nbit = abs(BITPIX)*GCOUNT*(PCOUNT + Nelements); // GCOUNT and PCOUNT are defined // into the header NSLog(@"TOTAL NUMBER OF BIT [%i]\n",Nbit); int i=0; //FILL THE ARRAY double Value; for(int bit=0; bit < Nbit; bit+=sizeof(double)) { [Img getBytes:&Value range:NSMakeRange(bit,sizeof(double))]; NSLog(@"[%i]:(%u)%.8G\n",i,bit,Value); (*array)[i]=Value; i++; } return (*array); } However, the value I print in the loop are very different from the expected values (compared using official FITS software). Therefore, I think that the Obj-C double does not use the IEEE-754 convention as well as the Obj-C int are not twos-complement. I am really not familiar with this two convention (IEEE and twos-complement) and would like to know how I can do this conversion with Obj-C. In advance many thanks for any help or information.

    Read the article

  • web grid server pagination trigger multiple controller call when changing page

    - by Thomas Scattolin
    When I server-filter on "au" my web grid and change page, multiple call to the controller are done : the first with 0 filtering, the second with "a" filtering, the third with "au" filtering. My table load huge data so the first call is longer than others. I see the grid displaying firstly the third call result, then the second, and finally the first call (this order correspond to the response time of my controller due to filter parameter) Why are all that controller call made ? Can't just my controller be called once with my total filter "au" ? What should I do ? Here is my grid : $("#" + gridId).kendoGrid({ selectable: "row", pageable: true, filterable:true, scrollable : true, //scrollable: { // virtual: true //false // Bug : Génère un affichage multiple... //}, navigatable: true, groupable: true, sortable: { mode: "multiple", // enables multi-column sorting allowUnsort: true }, dataSource: { type: "json", serverPaging: true, serverSorting: true, serverFiltering: true, serverGrouping:false, // Ne fonctionne pas... pageSize: '@ViewBag.Pagination', transport: { read: { url: Procvalue + "/LOV", type: "POST", dataType: "json", contentType: "application/json; charset=utf-8" }, parameterMap: function (options, type) { // Mise à jour du format d'envoi des paramètres // pour qu'ils puissent être correctement interprétés côté serveur. // Construction du paramètre sort : if (options.sort != null) { var sort = options.sort; var sort2 = ""; for (i = 0; i < sort.length; i++) { sort2 = sort2 + sort[i].field + '-' + sort[i].dir + '~'; } options.sort = sort2; } if (options.group != null) { var group = options.group; var group2 = ""; for (i = 0; i < group.length; i++) { group2 = group2 + group[i].field + '-' + group[i].dir + '~'; } options.group = group2; } if (options.filter != null) { var filter = options.filter.filters; var filter2 = ""; for (i = 0; i < filter.length; i++) { // Vérification si type colonne == string. // Parcours des colonnes pour trouver celle qui a le même nom de champ. var type = ""; for (j = 0 ; j < colonnes.length ; j++) { if (colonnes[j].champ == filter[i].field) { type = colonnes[j].type; break; } } if (filter2.length == 0) { if (type == "string") { // Avec '' autour de la valeur. filter2 = filter2 + filter[i].field + '~' + filter[i].operator + "~'" + filter[i].value + "'"; } else { // Sans '' autour de la valeur. filter2 = filter2 + filter[i].field + '~' + filter[i].operator + "~" + filter[i].value; } } else { if (type == "string") { // Avec '' autour de la valeur. filter2 = filter2 + '~' + options.filter.logic + '~' + filter[i].field + '~' + filter[i].operator + "~'" + filter[i].value + "'"; }else{ filter2 = filter2 + '~' + options.filter.logic + '~' + filter[i].field + '~' + filter[i].operator + "~" + filter[i].value; } } } options.filter = filter2; } var json = JSON.stringify(options); return json; } }, schema: { data: function (data) { return eval(data.data.Data); }, total: function (data) { return eval(data.data.Total); } }, filter: { logic: "or", filters:filtre(valeur) } }, columns: getColonnes(colonnes) }); Here is my controller : [HttpPost] public ActionResult LOV([DataSourceRequest] DataSourceRequest request) { return Json(CProduitsManager.GetProduits().ToDataSourceResult(request)); }

    Read the article

  • Java thread dump where main thread has no call stack? (jsvc)

    - by dwhsix
    We have a java process running as a daemon (under jsvc). Every several days it just stops doing any work; output to the logfile stops (it is pretty verbose, on 5-minute intervals) and it consumes no CPU or IO. There are no exceptions logged in the logfile nor in syserr or sysout. The last log statement is just prior to a db commit being done, but there is no open connection on the db server (MySQL) and reviewing the code, there should always be additional log output after that, even if it had encountered an exception that was going to bubble up. The most curious thing I find is that in the thread dump (included below), there's no thread in our code at all, and the main thread seems to have no context whatsoever: "main" prio=10 tid=0x0000000000614000 nid=0x445d runnable [0x0000000000000000] java.lang.Thread.State: RUNNABLE As noted earlier, this is a daemon process running using jsvc, but I don't know if that has anything to do with it (I can restructure the code to also allow running it directly, to test). Any suggestions on what might be happening here? Thanks... dwh Full thread dump: Full thread dump Java HotSpot(TM) 64-Bit Server VM (14.2-b01 mixed mode): "MySQL Statement Cancellation Timer" daemon prio=10 tid=0x00002aaaf81b8800 nid=0x447b in Object.wait() [0x00002aaaf6a22000] java.lang.Thread.State: WAITING (on object monitor) at java.lang.Object.wait(Native Method) - waiting on <0x00002aaab5556d50> (a java.util.TaskQueue) at java.lang.Object.wait(Object.java:485) at java.util.TimerThread.mainLoop(Timer.java:483) - locked <0x00002aaab5556d50> (a java.util.TaskQueue) at java.util.TimerThread.run(Timer.java:462) "Low Memory Detector" daemon prio=10 tid=0x00000000006a4000 nid=0x4479 runnable [0x0000000000000000] java.lang.Thread.State: RUNNABLE "CompilerThread1" daemon prio=10 tid=0x00000000006a1000 nid=0x4477 waiting on condition [0x0000000000000000] java.lang.Thread.State: RUNNABLE "CompilerThread0" daemon prio=10 tid=0x000000000069d000 nid=0x4476 waiting on condition [0x0000000000000000] java.lang.Thread.State: RUNNABLE "Signal Dispatcher" daemon prio=10 tid=0x000000000069b000 nid=0x4465 waiting on condition [0x0000000000000000] java.lang.Thread.State: RUNNABLE "Finalizer" daemon prio=10 tid=0x0000000000678800 nid=0x4464 in Object.wait() [0x00002aaaf61d6000] java.lang.Thread.State: WAITING (on object monitor) at java.lang.Object.wait(Native Method) - waiting on <0x00002aaab54a1cb8> (a java.lang.ref.ReferenceQueue$Lock) at java.lang.ref.ReferenceQueue.remove(ReferenceQueue.java:118) - locked <0x00002aaab54a1cb8> (a java.lang.ref.ReferenceQueue$Lock) at java.lang.ref.ReferenceQueue.remove(ReferenceQueue.java:134) at java.lang.ref.Finalizer$FinalizerThread.run(Finalizer.java:159) "Reference Handler" daemon prio=10 tid=0x0000000000676800 nid=0x4463 in Object.wait() [0x00002aaaf60d5000] java.lang.Thread.State: WAITING (on object monitor) at java.lang.Object.wait(Native Method) - waiting on <0x00002aaab54a1cf0> (a java.lang.ref.Reference$Lock) at java.lang.Object.wait(Object.java:485) at java.lang.ref.Reference$ReferenceHandler.run(Reference.java:116) - locked <0x00002aaab54a1cf0> (a java.lang.ref.Reference$Lock) "main" prio=10 tid=0x0000000000614000 nid=0x445d runnable [0x0000000000000000] java.lang.Thread.State: RUNNABLE "VM Thread" prio=10 tid=0x0000000000670000 nid=0x4462 runnable "GC task thread#0 (ParallelGC)" prio=10 tid=0x000000000061e000 nid=0x445e runnable "GC task thread#1 (ParallelGC)" prio=10 tid=0x0000000000620000 nid=0x445f runnable "GC task thread#2 (ParallelGC)" prio=10 tid=0x0000000000622000 nid=0x4460 runnable "GC task thread#3 (ParallelGC)" prio=10 tid=0x0000000000623800 nid=0x4461 runnable "VM Periodic Task Thread" prio=10 tid=0x00000000006a6800 nid=0x447a waiting on condition JNI global references: 797 Heap PSYoungGen total 162944K, used 48388K [0x00002aaadff40000, 0x00002aaaf2ab0000, 0x00002aaaf5490000) eden space 102784K, 47% used [0x00002aaadff40000,0x00002aaae2e81170,0x00002aaae63a0000) from space 60160K, 0% used [0x00002aaaeb850000,0x00002aaaeb850000,0x00002aaaef310000) to space 86720K, 0% used [0x00002aaae63a0000,0x00002aaae63a0000,0x00002aaaeb850000) PSOldGen total 699072K, used 699072K [0x00002aaab5490000, 0x00002aaadff40000, 0x00002aaadff40000) object space 699072K, 100% used [0x00002aaab5490000,0x00002aaadff40000,0x00002aaadff40000) PSPermGen total 21248K, used 9252K [0x00002aaab0090000, 0x00002aaab1550000, 0x00002aaab5490000) object space 21248K, 43% used [0x00002aaab0090000,0x00002aaab09993e8,0x00002aaab1550000)

    Read the article

  • Java Refuses to Start - Could not reserve enough space for object heap

    - by Randyaa
    Background We have a pool of aproximately 20 linux blades. Some are running Suse, some are running Redhat. ALL share NAS space which contains the following 3 folders: /NAS/app/java - a symlink that points to an installation of a Java JDK. Currently version 1.5.0_10 /NAS/app/lib - a symlink that points to a version of our application. /NAS/data - directory where our output is written All our machines have 2 processors (hyperthreaded) with 4gb of physical memory and 4gb of swap space. We limit the number of 'jobs' each machine can process at a given time to 6 (this number likely needs to change, but that does not enter into the current problem so please ignore it for the time being). Some of our jobs set a Max Heap size of 512mb, some others reserve a Max Heap size of 2048mb. Again, we realize we could go over our available memory if 6 jobs started on the same machine with the heap size set to 2048, but to our knowledge this has not yet occurred. The Problem Once and a while a Job will fail immediately with the following message: Error occurred during initialization of VM Could not reserve enough space for object heap Could not create the Java virtual machine. We used to chalk this up to too many jobs running at the same time on the same machine. The problem happened infrequently enough (MAYBE once a month) that we'd just restart it and everything would be fine. The problem has recently gotten much worse. All of our jobs which request a max heap size of 2048m fail immediately almost every time and need to get restarted several times before completing. We've gone out to individual machines and tried executing them manually with the same result. Debugging It turns out that the problem only exists for our SuSE boxes. The reason it has been happening more frequently is becuase we've been adding more machines, and the new ones are SuSE. 'cat /proc/version' on the SuSE boxes give us: Linux version 2.6.5-7.244-bigsmp (geeko@buildhost) (gcc version 3.3.3 (SuSE Linux)) #1 SMP Mon Dec 12 18:32:25 UTC 2005 'cat /proc/version' on the RedHat boxes give us: Linux version 2.4.21-32.0.1.ELsmp ([email protected]) (gcc version 3.2.3 20030502 (Red Hat Linux 3.2.3-52)) #1 SMP Tue May 17 17:52:23 EDT 2005 'uname -a' gives us the following on BOTH types of machines: UTC 2005 i686 i686 i386 GNU/Linux No jobs are running on the machine, and no other processes are utilizing much memory. All of the processes currently running might be using 100mb total. 'top' currently shows the following: Mem: 4146528k total, 3536360k used, 610168k free, 132136k buffers Swap: 4194288k total, 0k used, 4194288k free, 3283908k cached 'vmstat' currently shows the following: procs -----------memory---------- ---swap-- -----io---- --system-- ----cpu---- r b swpd free buff cache si so bi bo in cs us sy id wa 0 0 0 610292 132136 3283908 0 0 0 2 26 15 0 0 100 0 If we kick off a job with the following command line (Max Heap of 1850mb) it starts fine: java/bin/java -Xmx1850M -cp helloworld.jar HelloWorld Hello World If we bump up the max heap size to 1875mb it fails: java/bin/java -Xmx1875M -cp helloworld.jar HelloWorld Error occurred during initialization of VM Could not reserve enough space for object heap Could not create the Java virtual machine. It's quite clear that the memory currently being used is for Buffering/Caching and that's why so little is being displayed as 'free'. What isn't clear is why there is a magical 1850mb line where anything higher means Java can't start. Any explanations would be greatly appreciated.

    Read the article

  • my webserver with 16GB ram shows all RAM as used, but is it really, see the 'top'

    - by Alex
    I have some questions about my web server. Its a LAMP web server running centos 5.5 and php5, mysql5. The server gets hundreds (maybe thousand) of concurrent users during peak hours. I'm trying to optimize a little and understand "top". From what I can see: all 16GB of my ram have been used up? does that mean that my server needs more memory? My swap is only 2GB, should it be increased? usually during peak hours my server load average first number is about 2.5-3. What could I do to optimize the server so that the load average even during peak doesn't go above 1? In the past I was told a good working server should stay under 1 load, is this still true? Although even during load of 2.5-3, server pages and applications seem to load with pretty good speed. what should the memory size in php.ini be set to? top - 14:30:18 up 2 days, 12:41, 5 users, load average: 1.25, 1.74, 2.92 Tasks: 305 total, 2 running, 302 sleeping, 0 stopped, 1 zombie Cpu(s): 6.3%us, 0.9%sy, 0.0%ni, 92.5%id, 0.2%wa, 0.0%hi, 0.1%si, 0.0%st Mem: 16427200k total, 16111472k used, 315728k free, 3120316k buffers Swap: 2104496k total, 268k used, 2104228k free, 6216756k cached PID USER PR NI VIRT RES SHR S %CPU %MEM TIME+ COMMAND 29080 apache 15 0 358m 36m 5192 S 20.2 0.2 2:08.40 httpd 29093 apache 18 0 357m 36m 5192 S 18.2 0.2 2:02.52 httpd 29079 apache 15 0 370m 49m 5832 S 10.0 0.3 2:32.14 httpd 1812 apache 15 0 370m 49m 5196 S 7.3 0.3 2:25.30 httpd 5204 apache 15 0 358m 36m 5168 S 5.3 0.2 0:59.28 httpd 29075 apache 15 0 370m 48m 5184 S 3.3 0.3 2:15.93 httpd 9712 apache 15 0 360m 38m 5180 S 3.0 0.2 0:54.81 httpd 29072 apache 16 0 358m 36m 5192 S 2.7 0.2 2:24.43 httpd 6310 apache 17 0 388m 67m 5180 S 2.3 0.4 0:58.85 httpd 8674 apache 15 0 343m 21m 4980 S 2.0 0.1 0:07.91 httpd 29085 apache 15 0 371m 49m 5224 S 2.0 0.3 2:16.86 httpd 29083 apache 15 0 370m 48m 5196 S 1.7 0.3 2:10.64 httpd 5575 apache 15 0 357m 36m 5228 S 1.3 0.2 0:53.78 httpd 29066 apache 15 0 379m 59m 5860 R 1.3 0.4 2:11.93 httpd 29078 apache 15 0 370m 48m 5188 S 1.3 0.3 2:14.52 httpd 29084 apache 15 0 370m 48m 5208 S 1.0 0.3 2:02.49 httpd 29089 apache 15 0 370m 48m 5188 S 1.0 0.3 2:27.61 httpd 29082 apache 15 0 390m 68m 5188 S 0.7 0.4 2:32.48 httpd 29984 apache 15 0 358m 36m 5228 S 0.7 0.2 2:08.32 httpd 3571 root 16 0 13400 1792 848 S 0.3 0.0 2:37.89 top 4419 mysql 15 0 668m 175m 7204 S 0.3 1.1 3:32.25 mysqld 28181 root 15 0 90460 3624 2680 S 0.3 0.0 0:17.60 sshd 29091 apache 15 0 390m 69m 5196 S 0.3 0.4 2:29.99 httpd 32476 root 15 0 12900 1320 848 R 0.3 0.0 0:06.46 top 1 root 15 0 10372 680 572 S 0.0 0.0 0:02.01 init 2 root RT -5 0 0 0 S 0.0 0.0 0:00.51 migration/0 3 root 34 19 0 0 0 S 0.0 0.0 0:00.07 ksoftirqd/0 4 root RT -5 0 0 0 S 0.0 0.0 0:00.00 watchdog/0 5 root RT -5 0 0 0 S 0.0 0.0 0:00.12 migration/1 6 root 34 19 0 0 0 S 0.0 0.0 0:00.03 ksoftirqd/1 7 root RT -5 0 0 0 S 0.0 0.0 0:00.00 watchdog/1 8 root RT -5 0 0 0 S 0.0 0.0 0:00.06 migration/2 9 root 34 19 0 0 0 S 0.0 0.0 0:00.03 ksoftirqd/2 10 root RT -5 0 0 0 S 0.0 0.0 0:00.00 watchdog/2 11 root RT -5 0 0 0 S 0.0 0.0 0:00.06 migration/3 12 root 34 19 0 0 0 S 0.0 0.0 0:00.04 ksoftirqd/3 13 root RT -5 0 0 0 S 0.0 0.0 0:00.00 watchdog/3 14 root RT -5 0 0 0 S 0.0 0.0 0:01.45 migration/4 15 root 34 19 0 0 0 S 0.0 0.0 0:00.01 ksoftirqd/4 16 root RT -5 0 0 0 S 0.0 0.0 0:00.00 watchdog/4 17 root RT -5 0 0 0 S 0.0 0.0 0:00.22 migration/5 18 root 34 19 0 0 0 S 0.0 0.0 0:00.01 ksoftirqd/5 19 root RT -5 0 0 0 S 0.0 0.0 0:00.00 watchdog/5 20 root RT -5 0 0 0 S 0.0 0.0 0:00.15 migration/6 21 root 34 19 0 0 0 S 0.0 0.0 0:00.02 ksoftirqd/6 22 root RT -5 0 0 0 S 0.0 0.0 0:00.00 watchdog/6 23 root RT -5 0 0 0 S 0.0 0.0 0:00.15 migration/7 24 root 34 19 0 0 0 S 0.0 0.0 0:00.01 ksoftirqd/7 25 root RT -5 0 0 0 S 0.0 0.0 0:00.00 watchdog/7 26 root RT -5 0 0 0 S 0.0 0.0 0:00.19 migration/8 27 root 34 19 0 0 0 S 0.0 0.0 0:00.04 ksoftirqd/8 28 root RT -5 0 0 0 S 0.0 0.0 0:00.00 watchdog/8 29 root RT -5 0 0 0 S 0.0 0.0 0:00.34 migration/9 30 root 34 19 0 0 0 S 0.0 0.0 0:00.03 ksoftirqd/9 31 root RT -5 0 0 0 S 0.0 0.0 0:00.00 watchdog/9 32 root RT -5 0 0 0 S 0.0 0.0 0:00.16 migration/10 33 root 34 19 0 0 0 S 0.0 0.0 0:00.04 ksoftirqd/10 34 root RT -5 0 0 0 S 0.0 0.0 0:00.00 watchdog/10 35 root RT -5 0 0 0 S 0.0 0.0 0:00.12 migration/11 36 root 34 19 0 0 0 S 0.0 0.0 0:00.05 ksoftirqd/11 37 root RT -5 0 0 0 S 0.0 0.0 0:00.00 watchdog/11 38 root RT -5 0 0 0 S 0.0 0.0 0:00.35 migration/12

    Read the article

  • Linq2SQL vs NHibernate performance (have I gone mad?)

    - by HeavyWave
    I have written the following tests to compare performance of Linq2SQL and NHibernate and I find results to be somewhat strange. Mappings are straight forward and identical for both. Both are running against a live DB. Although I'm not deleting Campaigns in case of Linq, but that shouldn't affect performance by more than 10 ms. Linq: [Test] public void Test1000ReadsWritesToAgentStateLinqPrecompiled() { Stopwatch sw = new Stopwatch(); Stopwatch swIn = new Stopwatch(); sw.Start(); for (int i = 0; i < 1000; i++) { swIn.Reset(); swIn.Start(); ReadWriteAndDeleteAgentStateWithLinqPrecompiled(); swIn.Stop(); Console.WriteLine("Run ReadWriteAndDeleteAgentState: " + swIn.ElapsedMilliseconds + " ms"); } sw.Stop(); Console.WriteLine("Total Time: " + sw.ElapsedMilliseconds + " ms"); Console.WriteLine("Average time to execute queries: " + sw.ElapsedMilliseconds / 1000 + " ms"); } private static readonly Func<AgentDesktop3DataContext, int, EntityModel.CampaignDetail> GetCampaignById = CompiledQuery.Compile<AgentDesktop3DataContext, int, EntityModel.CampaignDetail>( (ctx, sessionId) => (from cd in ctx.CampaignDetails join a in ctx.AgentCampaigns on cd.CampaignDetailId equals a.CampaignDetailId where a.AgentStateId == sessionId select cd).FirstOrDefault()); private void ReadWriteAndDeleteAgentStateWithLinqPrecompiled() { int id = 0; using (var ctx = new AgentDesktop3DataContext()) { EntityModel.AgentState agentState = new EntityModel.AgentState(); var campaign = new EntityModel.CampaignDetail { CampaignName = "Test" }; var campaignDisposition = new EntityModel.CampaignDisposition { Code = "123" }; campaignDisposition.Description = "abc"; campaign.CampaignDispositions.Add(campaignDisposition); agentState.CallState = 3; campaign.AgentCampaigns.Add(new AgentCampaign { AgentState = agentState }); ctx.CampaignDetails.InsertOnSubmit(campaign); ctx.AgentStates.InsertOnSubmit(agentState); ctx.SubmitChanges(); id = agentState.AgentStateId; } using (var ctx = new AgentDesktop3DataContext()) { var dbAgentState = ctx.GetAgentStateById(id); Assert.IsNotNull(dbAgentState); Assert.AreEqual(dbAgentState.CallState, 3); var campaignDetails = GetCampaignById(ctx, id); Assert.AreEqual(campaignDetails.CampaignDispositions[0].Description, "abc"); } using (var ctx = new AgentDesktop3DataContext()) { ctx.DeleteSessionById(id); } } NHibernate (the loop is the same): private void ReadWriteAndDeleteAgentState() { var id = WriteAgentState().Id; StartNewTransaction(); var dbAgentState = agentStateRepository.Get(id); Assert.IsNotNull(dbAgentState); Assert.AreEqual(dbAgentState.CallState, 3); Assert.AreEqual(dbAgentState.Campaigns[0].Dispositions[0].Description, "abc"); var campaignId = dbAgentState.Campaigns[0].Id; agentStateRepository.Delete(dbAgentState); NHibernateSession.Current.Transaction.Commit(); Cleanup(campaignId); NHibernateSession.Current.BeginTransaction(); } Results: NHibernate: Total Time: 9469 ms Average time to execute 13 queries: 9 ms Linq: Total Time: 127200 ms Average time to execute 13 queries: 127 ms Linq lost by 13.5 times! Event with precompiled queries (both read queries are precompiled). This can't be right, although I expected NHibernate to be faster, this is just too big of a difference, considering mappings are identical and NHibernate actually executes more queries against the DB.

    Read the article

  • Why is my mdadm raid-1 recovery so slow?

    - by dimmer
    On a system I'm running Ubuntu 10.04. My raid-1 restore started out fast but quickly became ridiculously slow (at this rate the restore will take 150 days!): dimmer@paimon:~$ cat /proc/mdstat Personalities : [linear] [multipath] [raid0] [raid1] [raid6] [raid5] [raid4] [raid10] md0 : active raid1 sdc1[2] sdb1[1] 1953513408 blocks [2/1] [_U] [====>................] recovery = 24.4% (477497344/1953513408) finish=217368.0min speed=113K/sec unused devices: <none> Eventhough I have set the kernel variables to reasonably quick values: dimmer@paimon:~$ cat /proc/sys/dev/raid/speed_limit_min 1000000 dimmer@paimon:~$ cat /proc/sys/dev/raid/speed_limit_max 100000000 I am using 2 2.0TB Western Digital Hard Disks, WDC WD20EARS-00M and WDC WD20EARS-00J. I believe they have been partitioned such that their sectors are aligned. dimmer@paimon:/sys$ sudo parted /dev/sdb GNU Parted 2.2 Using /dev/sdb Welcome to GNU Parted! Type 'help' to view a list of commands. (parted) p Model: ATA WDC WD20EARS-00M (scsi) Disk /dev/sdb: 2000GB Sector size (logical/physical): 512B/512B Partition Table: gpt Number Start End Size File system Name Flags 1 1049kB 2000GB 2000GB ext4 (parted) unit s (parted) p Number Start End Size File system Name Flags 1 2048s 3907028991s 3907026944s ext4 (parted) q dimmer@paimon:/sys$ sudo parted /dev/sdc GNU Parted 2.2 Using /dev/sdc Welcome to GNU Parted! Type 'help' to view a list of commands. (parted) p Model: ATA WDC WD20EARS-00J (scsi) Disk /dev/sdc: 2000GB Sector size (logical/physical): 512B/4096B Partition Table: gpt Number Start End Size File system Name Flags 1 1049kB 2000GB 2000GB ext4 I am beginning to think that I have a hardware problem, otherwise I can't imagine why the mdadm restore should be so slow. I have done a benchmark on /dev/sdc using Ubuntu's disk utility GUI app, and the results looked normal so I know that sdc has the capability to write faster than this. I also had the same problem on a similar WD drive that I RMAd because of bad sectors. I suppose it's possible they sent me a replacement with bad sectors too, although there are no SMART values showing them yet. Any ideas? Thanks. As requested, output of top sorted by cpu usage (notice there is ~0 cpu usage). iowait is also zero which seems strange: top - 11:35:13 up 2 days, 9:40, 3 users, load average: 2.87, 2.58, 2.30 Tasks: 142 total, 1 running, 141 sleeping, 0 stopped, 0 zombie Cpu(s): 0.0%us, 0.2%sy, 0.0%ni, 99.8%id, 0.0%wa, 0.0%hi, 0.0%si, 0.0%st Mem: 3096304k total, 1482164k used, 1614140k free, 617672k buffers Swap: 1526132k total, 0k used, 1526132k free, 535416k cached PID USER PR NI VIRT RES SHR S %CPU %MEM TIME+ COMMAND 45 root 20 0 0 0 0 S 0 0.0 2:17.02 scsi_eh_0 1 root 20 0 2808 1752 1204 S 0 0.1 0:00.46 init 2 root 20 0 0 0 0 S 0 0.0 0:00.00 kthreadd 3 root RT 0 0 0 0 S 0 0.0 0:00.02 migration/0 4 root 20 0 0 0 0 S 0 0.0 0:00.17 ksoftirqd/0 5 root RT 0 0 0 0 S 0 0.0 0:00.00 watchdog/0 6 root RT 0 0 0 0 S 0 0.0 0:00.02 migration/1 ... dmesg errors, definitely looking like hardware: [202884.000157] ata5.00: exception Emask 0x0 SAct 0x0 SErr 0x0 action 0x6 frozen [202884.007015] ata5.00: failed command: FLUSH CACHE EXT [202884.013728] ata5.00: cmd ea/00:00:00:00:00/00:00:00:00:00/a0 tag 0 [202884.013730] res 40/00:00:ff:59:2e/00:00:35:00:00/e0 Emask 0x4 (timeout) [202884.033667] ata5.00: status: { DRDY } [202884.040329] ata5: hard resetting link [202889.400050] ata5: link is slow to respond, please be patient (ready=0) [202894.048087] ata5: COMRESET failed (errno=-16) [202894.054663] ata5: hard resetting link [202899.412049] ata5: link is slow to respond, please be patient (ready=0) [202904.060107] ata5: COMRESET failed (errno=-16) [202904.066646] ata5: hard resetting link [202905.840056] ata5: SATA link up 3.0 Gbps (SStatus 123 SControl 300) [202905.849178] ata5.00: configured for UDMA/133 [202905.849188] ata5: EH complete [203899.000292] ata5.00: exception Emask 0x0 SAct 0x0 SErr 0x0 action 0x6 frozen [203899.007096] ata5.00: failed command: IDENTIFY DEVICE [203899.013841] ata5.00: cmd ec/00:01:00:00:00/00:00:00:00:00/00 tag 0 pio 512 in [203899.013843] res 40/00:00:ff:f9:f6/00:00:38:00:00/e0 Emask 0x4 (timeout) [203899.041232] ata5.00: status: { DRDY } [203899.048133] ata5: hard resetting link [203899.816134] ata5: SATA link up 3.0 Gbps (SStatus 123 SControl 300) [203899.826062] ata5.00: configured for UDMA/133 [203899.826079] ata5: EH complete [204375.000200] ata5.00: exception Emask 0x0 SAct 0x0 SErr 0x0 action 0x6 frozen [204375.007421] ata5.00: failed command: IDENTIFY DEVICE [204375.014799] ata5.00: cmd ec/00:01:00:00:00/00:00:00:00:00/00 tag 0 pio 512 in [204375.014800] res 40/00:00:ff:0c:0f/00:00:39:00:00/e0 Emask 0x4 (timeout) [204375.044374] ata5.00: status: { DRDY } [204375.051842] ata5: hard resetting link [204380.408049] ata5: link is slow to respond, please be patient (ready=0) [204384.440076] ata5: SATA link up 3.0 Gbps (SStatus 123 SControl 300) [204384.449938] ata5.00: configured for UDMA/133 [204384.449955] ata5: EH complete [204395.988135] ata5.00: exception Emask 0x0 SAct 0x0 SErr 0x0 action 0x6 frozen [204395.988140] ata5.00: failed command: IDENTIFY DEVICE [204395.988147] ata5.00: cmd ec/00:01:00:00:00/00:00:00:00:00/00 tag 0 pio 512 in [204395.988149] res 40/00:00:ff:0c:0f/00:00:39:00:00/e0 Emask 0x4 (timeout) [204395.988151] ata5.00: status: { DRDY } [204395.988156] ata5: hard resetting link [204399.320075] ata5: SATA link up 3.0 Gbps (SStatus 123 SControl 300) [204399.330487] ata5.00: configured for UDMA/133 [204399.330503] ata5: EH complete

    Read the article

  • log-back and thirdparty writing to stdout. How to stop them getting interleaved.

    - by David Roussel
    First some background. I have a batch-type java process run from a DOS batch script. All the java logging goes to stdout, and the batch script redirects the stdout to a file. (This is good for me because I can ECHO from the script and it gets into the log file, so I can see all the java JVM command line args, which is great for debugging.) I may not I use slf4j API, and for the backend I used to use log4j, but recently switched to logback-classic. Although all my application code uses slf4j, I have a third party library that does it's own logging (not using a standard API) which also gets written to stdout. The problem is that sometimes log lines get mixed up and don't cleanly appear on separate lines. Here is an example of some messed up output: 2010-05-28 18:00:44.783 [thread-1 ] INFO CreditCorrelationElementBuilderImpl - Bump parameters exist for scenario, now attempting bumping. [indexDisplayName=STANDARD_S1_v300] 2010-05-28 18:01:43.517 [thread-1 ] INFO CreditCorrelationElementBuilderImpl - Found adjusted point in data, now applying bump. [point=0.144040000000000] 2010-05-28 18:01:58.642 [thread-1 ] DEBUG com.company.request.Request - Generated request for [dealName=XXX_20050225_01[5],dealType=GENERIC_XXX,correlationType=2,copulaType=1] in 73.8 s, Simon Stopwatch: [sys1.batchpricer.reqgen.gen INHERIT] total 1049 s, counter 24, max 74.1 s, min 212 ms 2010-05-28 18:05/28/10 18:02:20.236 INFO: [ServiceEvent] SubmittedTask:BC-STRESS_04_FZBC-2010-05-21-545024448189310126-23 01:58.658 [req-writer-2b ] INFO .c.g.r.o.OptionalFileDocumentOutput - Writing request XML to \\filserver\dir\file1.xml - write time: 21.4 ms - Simon Stopwatch: [sys1.batchpricer.reqgen.writeinputfile INHERIT] total 905 ms, counter 24, max 109 ms, min 10.8 ms 2010-05-28 18:02:33.626 [ResponseCallbacks-1: DriverJobSpace$TakeJobRunner$1] ERROR c.c.s.s.D.CalculatorCallback - Id:23 no deal found !! 2010-0505/28/10 18:02:50.267 INFO: [ServiceEvent] CompletedTask:BC-STRESS_04_FZBC-2010-05-21-545024448189310126-23:Total:24 Now comparing back to older log files, it seems the problem didn't occur when using log4j as the logging backend. So logback must be doing something different. The problem seems to be that although PrintStream.write(byte buf[], int off, int len) is synchronized, however I can see in ch.qos.logback.core.joran.spi.ConsoleTarget that System.out.write(int b) is the only write method called. So inbetween logback outputting each byte, the thirdparty library is managing to write a whole string to the stdout. (Not only is this cause me a problem, but it must also be a little inefficient?) Is there any other fix to this interleaving problem than patching the code to ConsoleTarget so it implments the other write methods? Any nice work arounds. Or should I just file a bug report? Here is my logback.xml: <configuration> <appender name="STDOUT" class="ch.qos.logback.core.ConsoleAppender"> <encoder> <pattern>%d{yyyy-MM-dd HH:mm:ss.SSS} [%-16thread] %-5level %-35.35logger{30} - %msg%n</pattern> </encoder> </appender> <root level="DEBUG"> <appender-ref ref="STDOUT" /> </root> </configuration> I'm using logback 0.9.20 with java 1.6.0_07.

    Read the article

  • Using IF statements to find string length in array for alignment (Visual Basic)

    - by Brodoin
    My question is just as it says in the title. How would one use IF statements to find the string-length of content in an array, and then make it so that they show up in a Rich Text Box with the left sides aligned? Noting that one value in my array is a Decimal. Imports System.IO Imports System.Convert Public Class frmAll 'Declare Streamreader Private objReader As StreamReader 'Declare arrays to hold the information Private strNumber(24) As String Private strName(24) As String Private strSize(24) As String Private decCost(24) As Integer Private Sub frmAll_Load(ByVal sender As System.Object, ByVal e As System.EventArgs) Handles MyBase.Load 'Set objReader objReader = New StreamReader("products.csv") 'Call the FillArray sub to fill the array Call FillArray() End Sub Private Sub FillArray() 'Declare variables and arrays Dim decCost(24, 1) As Decimal Dim strFields() As String Dim strRec As String Dim intCount As Integer = 0 Dim chrdelim As Char = ToChar(",") 'Set strRec to read the lines strRec = objReader.ReadLine 'Do while loop to fill array. Do While strRec <> Nothing strFields = strRec.Split(chrdelim) strNumber(intCount) = strFields(0) strName(intCount) = strFields(1) strSize(intCount) = strFields(2) decCost(intCount, 0) = ToDecimal(strFields(3)) decCost(intCount, 1) = ToDecimal(strFields(4)) 'Set strRec to read the lines again strRec = objReader.ReadLine 'increment the index intCount += 1 Loop 'Call the Calculate sub for calculation Call Calculate(decCost) End Sub Private Sub Calculate(ByVal numIn(,) As Decimal) 'Define arrays to hold total cost Dim decRowTotal(24) As Decimal 'Define variables to hold the counters for rows and columns Dim intR As Integer Dim intC As Integer 'Calcualte total cost For intC = 0 To 1 For intR = 0 To 24 decRowTotal(intR) += numIn(intR, intC) * 1 Next Next 'Call the Output sub to configure the output. Call Output(numIn, decRowTotal) End Sub Private Sub Output(ByVal NumIn(,) As Decimal, _ ByVal RowTotalIn() As Decimal) 'Variables Dim strOut As String Dim intR As Integer = 0 Dim intC As Integer = 0 'Set header for output. strOut = "ID" & vbTab & "Item" & vbTab & vbTab & vbTab & "Size" & _ vbTab & vbTab & vbTab & vbTab & "Total Price" & _ vbCrLf & "---------- ... -------------------------" & vbCrLf 'For loop to add each line to strOut, setting 'the RowTotalIn to currency. For intC = 0 To 24 strOut &= strNumber(intC) & vbTab strOut &= strName(intC) & vbTab strOut &= strSize(intC) & vbTab strOut &= RowTotalIn(intC).ToString("c") & vbCrLf Next 'Add strOut to rbtAll rtbAll.Text = strOut End Sub End Class Output It shows up with vbTabs in my output, but still, it looks similar in that they are not aligned. The first two do, but after that they are not, and I am totally lost. P0001 Coffee - Colombian Supreme 24/Case: Pre-Ground 1.75 Oz Bags $16.50 P0002 Coffee - Hazelnut 24/Case: Pre-Ground 1.75 Oz Bags $24.00 P0003 Coffee - Mild Blend 24/Case: Pre-Ground 1.75 Oz Bags $20.50 P0004 Coffee - Assorted Flavors 18/Case. Pre-Ground 1.75 Oz Bags $23.50 P0005 Coffee - Decaf 24/Case: Pre-Ground 1.75 Oz Bags $20.50

    Read the article

  • How to reduce virtual memory by optimising my PHP code?

    - by iCeR
    My current code (see below) uses 147MB of virtual memory! My provider has allocated 100MB by default and the process is killed once run, causing an internal error. The code is utilising curl multi and must be able to loop with more than 150 iterations whilst still minimizing the virtual memory. The code below is only set at 150 iterations and still causes the internal server error. At 90 iterations the issue does not occur. How can I adjust my code to lower the resource use / virtual memory? Thanks! <?php function udate($format, $utimestamp = null) { if ($utimestamp === null) $utimestamp = microtime(true); $timestamp = floor($utimestamp); $milliseconds = round(($utimestamp - $timestamp) * 1000); return date(preg_replace('`(?<!\\\\)u`', $milliseconds, $format), $timestamp); } $url = 'https://www.testdomain.com/'; $curl_arr = array(); $master = curl_multi_init(); for($i=0; $i<150; $i++) { $curl_arr[$i] = curl_init(); curl_setopt($curl_arr[$i], CURLOPT_URL, $url); curl_setopt($curl_arr[$i], CURLOPT_RETURNTRANSFER, 1); curl_setopt($curl_arr[$i], CURLOPT_SSL_VERIFYHOST, FALSE); curl_setopt($curl_arr[$i], CURLOPT_SSL_VERIFYPEER, FALSE); curl_multi_add_handle($master, $curl_arr[$i]); } do { curl_multi_exec($master,$running); } while($running > 0); for($i=0; $i<150; $i++) { $results = curl_multi_getcontent ($curl_arr[$i]); $results = explode("<br>", $results); echo $results[0]; echo "<br>"; echo $results[1]; echo "<br>"; echo udate('H:i:s:u'); echo "<br><br>"; usleep(100000); } ?> Processor Information Total processors: 8 Processor #1 Vendor GenuineIntel Name Intel(R) Xeon(R) CPU E5405 @ 2.00GHz Speed 1995.120 MHz Cache 6144 KB Processor #2 Vendor GenuineIntel Name Intel(R) Xeon(R) CPU E5405 @ 2.00GHz Speed 1995.120 MHz Cache 6144 KB Processor #3 Vendor GenuineIntel Name Intel(R) Xeon(R) CPU E5405 @ 2.00GHz Speed 1995.120 MHz Cache 6144 KB Processor #4 Vendor GenuineIntel Name Intel(R) Xeon(R) CPU E5405 @ 2.00GHz Speed 1995.120 MHz Cache 6144 KB Processor #5 Vendor GenuineIntel Name Intel(R) Xeon(R) CPU E5405 @ 2.00GHz Speed 1995.120 MHz Cache 6144 KB Processor #6 Vendor GenuineIntel Name Intel(R) Xeon(R) CPU E5405 @ 2.00GHz Speed 1995.120 MHz Cache 6144 KB Processor #7 Vendor GenuineIntel Name Intel(R) Xeon(R) CPU E5405 @ 2.00GHz Speed 1995.120 MHz Cache 6144 KB Processor #8 Vendor GenuineIntel Name Intel(R) Xeon(R) CPU E5405 @ 2.00GHz Speed 1995.120 MHz Cache 6144 KB Memory Information Memory for crash kernel (0x0 to 0x0) notwithin permissible range Memory: 8302344k/9175040k available (2176k kernel code, 80272k reserved, 901k data, 228k init, 7466304k highmem) System Information Linux server3.server.com 2.6.18-194.17.1.el5PAE #1 SMP Wed Sep 29 13:31:51 EDT 2010 i686 i686 i386 GNU/Linux Physical Disks SCSI device sda: 1952448512 512-byte hdwr sectors (999654 MB) sda: Write Protect is off sda: Mode Sense: 03 00 00 08 SCSI device sda: drive cache: write back SCSI device sda: 1952448512 512-byte hdwr sectors (999654 MB) sda: Write Protect is off sda: Mode Sense: 03 00 00 08 SCSI device sda: drive cache: write back sd 0:1:0:0: Attached scsi disk sda sd 4:0:0:0: Attached scsi removable disk sdb sd 0:1:0:0: Attached scsi generic sg4 type 0 sd 4:0:0:0: Attached scsi generic sg7 type 0 Current Memory Usage total used free shared buffers cached Mem: 8306672 7847384 459288 0 487912 6444548 -/+ buffers/cache: 914924 7391748 Swap: 4095992 496 4095496 Total: 12402664 7847880 4554784 Current Disk Usage Filesystem Size Used Avail Use% Mounted on /dev/mapper/VolGroup00-LogVol00 898G 307G 546G 36% / /dev/sda1 99M 19M 76M 20% /boot none 4.0G 0 4.0G 0% /dev/shm /var/tmpMnt 4.0G 1.8G 2.0G 48% /tmp

    Read the article

  • Creating a multidimensional array

    - by Jess McKenzie
    I have the following response and I was wanting to know how can I turn it into an multidimensional array foreach item [0][1] etc Controller $rece Response: array(16) { ["digital_delivery"]=> int(1) ["original_referrer"]=> string(11) "No Referrer" ["shop_rule_us_state_code"]=> string(1) "0" ["subtotal_ex_vat"]=> string(4) "9.99" ["subtotal_inc_vat"]=> string(4) "9.99" ["tax_amount"]=> string(4) "0.00" ["delivery_price"]=> string(4) "0.00" ["discount_deduction"]=> string(4) "0.00" ["currency_code"]=> string(3) "GBP" ["total"]=> string(4) "9.99" ["paid"]=> int(1) ["created"]=> string(19) "2013-10-31 21:03:44" ["website_id"]=> string(2) "64" ["first_name"]=> string(3) "Joe" ["last_name"]=> string(5) "Blogs" ["email"]=> string(17) "[email protected]" } array(16) { ["digital_delivery"]=> int(1) ["original_referrer"]=> string(11) "No Referrer" ["shop_rule_us_state_code"]=> string(1) "0" ["subtotal_ex_vat"]=> string(4) "9.99" ["subtotal_inc_vat"]=> string(4) "9.99" ["tax_amount"]=> string(4) "0.00" ["delivery_price"]=> string(4) "0.00" ["discount_deduction"]=> string(4) "0.00" ["currency_code"]=> string(3) "GBP" ["total"]=> string(4) "9.99" ["paid"]=> int(1) ["created"]=> string(19) "2013-10-31 21:03:44" ["website_id"]=> string(2) "64" ["first_name"]=> string(3) "Joe" ["last_name"]=> string(5) "Blogs" ["email"]=> string(13) "[email protected]" } array(16) { ["digital_delivery"]=> int(1) ["original_referrer"]=> string(11) "No Referrer" ["shop_rule_us_state_code"]=> string(1) "0" ["subtotal_ex_vat"]=> string(4) "9.99" ["subtotal_inc_vat"]=> string(4) "9.99" ["tax_amount"]=> string(4) "0.00" ["delivery_price"]=> string(4) "0.00" ["discount_deduction"]=> string(4) "0.00" ["currency_code"]=> string(3) "GBP" ["total"]=> string(4) "9.99" ["paid"]=> int(1) ["created"]=> string(19) "2013-10-31 21:03:44" ["website_id"]=> string(2) "64" ["first_name"]=> string(3) "Joe" ["last_name"]=> string(5) "Blogs" ["email"]=> string(15) "[email protected]" } Controller: foreach ($this->receivers as $rece) { $order_data['first_name'] = $rece[0]; $order_data['last_name'] = $rece[1]; $order_data['email'] = $rece[2]; $order_id = $this->orders_model->add_order_multi($order_data, $order_products_data); $this-receivers function: public function parse_receivers($receivers) { $this->receivers = explode( "\n", trim($receivers) ); $this->receivers = array_filter($this->receivers, 'trim'); $validReceivers = false; foreach($this->receivers as $key=>$receiver) { $validReceivers = true; $this->receivers[$key] = array_map( 'trim', explode(',', $receiver) ); if (count($this->receivers[$key]) != 3) { $line = $key + 1; $this->form_validation->set_message('parse_receivers', "There is an error in the %s at line $line ($receiver)"); return false; } } return $validReceivers; }

    Read the article

  • Why does my program not read full files?

    - by user593395
    I have written code in Java to read the content of a file. But it is working for small line of file only not for more than 1000 line of file. Please tell me me what error I have made in the below program. program: import java.io.DataInputStream; import java.io.DataOutputStream; import java.io.File; import java.io.FileInputStream; import java.io.FileNotFoundException; import java.io.FileOutputStream; import java.util.regex.Matcher; import java.util.regex.Pattern; public class aaru { public static void main(String args[]) throws FileNotFoundException { File sourceFile = new File("E:\\parser\\parse3.txt"); File destinationFile = new File("E:\\parser\\new.txt"); FileInputStream fileIn = new FileInputStream(sourceFile); FileOutputStream fileOut = new FileOutputStream(destinationFile); DataInputStream dataIn = new DataInputStream(fileIn); DataOutputStream dataOut = new DataOutputStream(fileOut); String str=""; String[] st; String sub[]=null; String word=""; String contents=""; String total=""; String stri="";; try { while((contents=dataIn.readLine())!=null) { total = contents.replaceAll(",",""); String str1=total.replaceAll("--",""); String str2=str1.replaceAll(";","" ); String str3=str2.replaceAll("&","" ); String str4=str3.replaceAll("^","" ); String str5=str4.replaceAll("#","" ); String str6=str5.replaceAll("!","" ); String str7=str6.replaceAll("/","" ); String str8=str7.replaceAll(":","" ); String str9=str8.replaceAll("]","" ); String str10=str9.replaceAll("\\?",""); String str11=str10.replaceAll("\\*",""); String str12=str11.replaceAll("\\'",""); Pattern pattern = Pattern.compile("\\s+", Pattern.CASE_INSENSITIVE | Pattern.DOTALL | Pattern.MULTILINE); Matcher matcher = pattern.matcher(str12); //boolean check = matcher.find(); String result=str12; Pattern p=Pattern.compile("^www\\.|\\@"); Matcher m=p.matcher(result); stri = m.replaceAll(" "); int i; int j; st=stri.split("\\."); for(i=0;i<st.length;i++) { st[i]=st[i].trim(); /*if(st[i].startsWith(" ")) st[i]=st[i].substring(1,st[i].length);*/ sub=st[i].split(" "); if(sub.length>1) { for(j=0;j<sub.length-1;j++) { word = word+sub[j]+","+sub[j+1]+"\r\n"; } } else { word = word+st[i]+"\r\n"; } } } System.out.println(word); dataOut.writeBytes(word+"\r\n"); fileIn.close(); fileOut.close(); dataIn.close(); dataOut.close(); } catch(Exception e) { System.out.print(e); } } }

    Read the article

< Previous Page | 59 60 61 62 63 64 65 66 67 68 69 70  | Next Page >